You are on page 1of 30

Capítulo 3

DINAMICA
DE UNA Dirac, Paul Adrien Maurice (1902-1984),

PARTICULA Físico teórico británico, nació en Bristol. Llegó a ser


Profesor Lucasiano en Cambridge Su teoría cuántica
relativista del electrón en movimiento le condujo en
1928 a postular la existencia del electrón positivo. La
teoría de Dirac fue confirmada en 1932, cuando el
físico norteamericano Carl Anderson descubrió al
positrón. En 1933 Dirac recibió el premio Nobel de
Física junto con el físico austriaco Erwin Schrodinger

De los tres fundadores de la moderna mecánica


cuántica, Heisenberg y Schrodinger pueden reclamar

Fuerza e interacciones 99 para sí el haber descubierto lo primeros asomos de la

Primera ley de Newton: Ley de la Inercia 99 teoría. Pero fue Dirac quien la puso junta y la reveló

Conceptos de fuerza y masa 100 completamente.

Segunda ley de Newton: ley fundamental de la La Ecuación de Dirac gobierna la mayor parte de la
dinámica 102
física y toda la química. Dirac vio cosas en términos
simples y claros, y no fue siempre claro para él
Tercera ley de Newton: acción y reacción 103
entender porque otras personas no veían las cosas en
Unidades de fuerza, masa y aceleración 104
términos similares. Esto ha llevado a un enorme
Masa y Peso 104
número de historias acerca de Dirac.
Marcos de referencia no inerciales 105
Nunca decía mucho, en contraste con su mujer pero
Algunas aplicaciones de las leyes de Newton 107
cuando hablaba, era de lo más provechoso el
Fuerzas de fricción 108
escucharlo.
Fuerzas y movimiento circular 110
Fuerzas de resistencia 112 Dirac ha hecho más que nadie en este siglo, con la
Fuerzas fundamentales 113 excepción de Einstein, para hacer avanzar la física y
Problemas resueltos 114 cambiar nuestra imagen del universo.
Problemas propuestos 136
68 Dinámica Daniel Fernández Palma / Raúl Zavala Sánchez

INTRODUCCION

La dinámica es la parte de la mecánica que estudia el En el caso mostrado en la figura 3.1, el efecto mecánico de
movimiento relacionado con las fuerzas que lo originan. La las fuerzas F1 y F2 es el mismo si: a) se cambia de A a 0 el
mecánica clásica es la teoría del movimiento basado en las punto de aplicación de F1. b) utilizamos sólo la resultante F
ideas de masa, fuerza y las leyes que relacionan estos en lugar de F1 y F2
conceptos físicos con las magnitudes cinemáticas: posición,
velocidad y aceleración. Fuerza resultante: F = F1 + F2

3.1 FUERZA E INTERACCIONES 3.2 PRIMERA LEY DE NEWTON: LEY DE LA INERCIA

El concepto de fuerza nos da una descripción cualitativa de la "Todo cuerpo mantiene su estado de reposo o de
interacción entre dos cuerpos o entre un cuerpo y su entorno movimiento rectilíneo uniforme a menos que una fuerza
exterior modifique dicho estado"
Un concepto riguroso de fuerza se dará a través del estudio
de las leyes de Newton. Por el momento daremos una idea Matemáticamente:
intuitiva de fuerza, basada en la experiencia diaria ⃗ = 0 ó a⃗ = 0
F (3.1)
Es muy común afirmar que hacemos fuerza al realizar ciertas Esta ley sintetiza tres ideas fundamentales:
operaciones tales como: tirar de una cuerda atada a un
cuerpo; empujar un automóvil para ponerlo en movimiento; 1) Toda fuerza es causa del movimiento o su presencia es
estirar o comprimir un resorte para provocar su deformación; necesaria para alterar el estado de reposo o de movimiento
caminar o patinar etc; en todos ellos lo hacemos venciendo
2) El estado de reposo o de movimiento rectilíneo uniforme
una fuerza que lo percibimos en nuestros músculos
son equivalentes. Para todo cuerpo material libre de fuerzas
Toda vez que una fuerza tenga lugar por contacto directo externas (sin interacciones con otros cuerpos) el reposo o el
entre dos cuerpos, la fuerza se denomina fuerza de contacto. movimiento uniforme son sus estados naturales.
Estas fuerzas son de naturaleza electromagnética y
3) Da la definición de un sistema de referencia inercial como
representan la suma total de un número muy grande de
aquel sistema de referencia desde el cual no existiendo
interacciones entre moléculas muy próximas entre si
ninguna fuerza externa que actúe sobre el cuerpo, se
Algunas fuerzas se manifiestan sin necesidad del contacto observaría la velocidad de éste exactamente constante
directo ya que actúan a distancia, aunque los cuerpos estén
Un punto de la superficie terrestre tomado como sistema de
separados, como ocurre en la atracción o repulsión de imanes
referencia no es inercial porque el movimiento combinado de
o en la fuerza gravitatoria que ejerce el Sol sobre los planetas
rotación y traslación de la Tierra en torno al Sol le comunica
para mantenerlos en su vecindad a pesar del continuo
a tal sistema un movimiento acelerado.
movimiento. La misma clase de fuerza es ejercida por la
Tierra sobre los cuerpos cercanos a su superficie ocasionando El Sol tampoco es un sistema de referencia inercial debido a
el peso de los mismos. A esta clase de fuerzas se les su movimiento alrededor del centro de la galaxia; Sin
denomina de largo alcance, embargo para fines prácticos, tanto la Tierra como el Sol o un
sistema fijo a estos se puede usar como sistema de referencia
Es decir que, en las actividades de la vida diaria a menudo
inercial al ser considerados despreciables los efectos de
empleamos y vemos actuar fuerzas en forma espontánea y
rotación y las interacciones mutuas. En este sentido, las
podemos concluir que la fuerza es causa de múltiples
estrellas constituyen sistemas de referencia inerciales.
fenómenos tales como el movimiento, el equilibrio, la
deformación de los cuerpos etc. EJEMPLO 3.1 Una esferilla de acero de radio muy pequeño
que se deja caer dentro del agua adquiere movimiento
Un sistema de fuerzas puede sustituirse por su resultante, la
rectilíneo uniforme casi inmediatamente después de soltada.
misma que se representa por una fuerza única como es el
caso de las fuerzas concurrentes o por una fuerza y un par
como es el caso de las fuerzas no concurrentes. En todos los
casos la resultante debe ser capaz de producir en el cuerpo, el
R
mismo efecto mecánico, que el conjunto de las fuerzas dadas.
En esta operación de composición es muy útil el principio de v
transmisibilidad de las fuerzas, el cual establece que el efecto
mecánico de una fuerza no varía si la fuerza se traslada a W
cualquier punto de su línea de acción, conservando su
módulo y sentido.

F2 F Figura 3.2 M.R.U. de una esferilla

En este caso no hay caída libre o movimiento acelerado


0 A F1 porque la fuerza peso W ha sido equilibrada o anulada por la
presencia de una fuerza de resistencia R que ofrece el agua.
De este modo se cumple la ecuación 3.1. Esto es:
Figura 3.1 Sólo una fuerza en lugar de dos F=W–R=0
Daniel Fernández Palma / Raúl Zavala Sánchez Dinámica 69

3.3 CONCEPTOS DE FUERZA Y MASA Veamos ahora la definición operacional de masa. Para ello
emplearemos un resorte calibrado para producir una fuerza
Según nuestras observaciones diarias los cuerpos cambian su determinada. Fijando el resorte a cuerpos distintos y
velocidad solamente por interacción con otros cuerpos. aplicando, para cada caso, la misma fuerza se observa que las
Describiremos esta interacción diciendo que los cuerpos aceleraciones producidas en los cuerpos son distintas.
ejercen fuerzas mutuas entre sí o que la interacción se mide
por una cantidad física llamada fuerza. 2m
La idea primaria que tenemos sobre fuerza, es la sensación de m
esfuerzo muscular que hacemos para deformar cualquier F a F 2a
objeto elástico, un resorte por ejemplo o para acelerar un
objeto.
Así tenemos la noción de dos efectos que puede producir una
fuerza aplicada a un cuerpo: efecto elástico o deformación de Figura 3.5 A menor masa la aceleración es mayor
un cuerpo fig 3.3a y efecto dinámico o aceleración del cuerpo
fig 3.3b Por ejemplo uniendo dos cuerpos idénticos y aplicando una
fuerza determinada, la aceleración que adquiere el conjunto
F a es exactamente mitad de la que produciría la misma fuerza
F actuando sobre uno sólo de los cuerpos. Se dice entonces que
en el primer caso hay más masa que en el segundo caso. En
x general se cumple que a menor masa del cuerpo corresponde
mayor aceleración. Esto nos sugiere que podemos cuantificar
(a) (b) el concepto de masa considerando la aceleración que una
fuerza determinada produce sobre distintos cuerpos.
Figura 3.3 Dos efectos de una fuerza Específicamente definiremos el cociente entre la masa de un
cuerpo y la de otro, como la razón inversa de las
aceleraciones producidas en ambos por la misma fuerza. Si
La deformación como la aceleración de un cuerpo depende una fuerza determinada produce la aceleración a1 cuando
de la dirección y de qué tan grande es la fuerza aplicada. Por actúa sobre el cuerpo (1) y la aceleración a2 al actuar sobre el
tanto la fuerza es una magnitud vectorial y en consecuencia cuerpo (2), el cociente de las masas de ambos cuerpos se
tiene todos los elementos que caracterizan a estas magnitudes define como:
Podemos suponer ahora que una fuerza se ejerce siempre por m 2 a1
intermedio de un resorte y que la dirección del resorte y su  (3.2)
deformación nos darán una indicación del vector fuerza. De m1 a 2
hecho diremos que dos fuerzas son iguales si producen
Se puede hallar la razón o el cociente de las masas de dos
iguales alargamientos en el mismo resorte y que si una fuerza
cuerpos cualesquiera aplicando la misma fuerza a ambos y
alarga un resorte en la misma cantidad que la producida por
comparando sus aceleraciones. Habiendo definido el cociente
la acción simultánea de dos fuerzas iguales a F actuando en
de las masas para dos cuerpos cualesquiera podemos establecer
el mismo sentido, esta fuerza es igual a 2F (fig 3.4) y así
una escala de masas escogiendo un cuerpo concreto como
sucesivamente podemos graduar los alargamientos del resorte
masa estándar y considerándolo arbitrariamente como unidad
en función de F.
de masa.
La cantidad m1a1 = m2a2 obtenida de la ecuación (3.2) se
toma como la medida cuantitativa de la fuerza (F). De
acuerdo con esto, el concepto de fuerza queda definido por la
ecuación siguiente:
F = ma (3.3)
La fuerza que actúa sobre una partícula es el producto de la
masa de éste por la aceleración producida por su interacción
con otros cuerpos. Esto es en esencia el significado de la
F segunda ley de Newton, que explicamos en la siguiente
sección.
F F
EJEMPLO 3.2 ¿Qué fuerza se necesita para que una masa de
10 kg acelere a razón de 5 m/s2 ?
2F
Solución. Aplicamos la ecuación (3.3)
Figura 3.4 El estiramiento es proporcional a la fuerza F = ma = (10 kg)(5 m/s2) = 50 kg.m/s2

Inversamente una fuerza desconocida podrá ahora ser medida 3.4 SEGUNDA LEY DE NEWTON: LEY
(el resorte así calibrado se denomina dinamómetro), Todo FUNDAMENTAL DE LA DINAMICA
esto nos permite escoger una unidad de fuerza y darle un
nombre arbitrario. Cabe recalcar que estamos definiendo la La Ecuación (3.3) es el enunciado matemático de la segunda
fuerza por la descripción de un método para su medida, esto ley del movimiento de Newton. El propio Newton expresó la
es, una definición operacional. ley como sigue:
70 Dinámica Daniel Fernández Palma / Raúl Zavala Sánchez

"La variación del movimiento es proporcional a la fuerza dice que la inercia es pequeña. Por tanto la masa de un
motriz aplicada y tiene lugar en la dirección de la recta cuerpo representa la medida de su inercia o propiedad que
sobre la cual se aplica dicha fuerza" tienen los cuerpos de ofrecer resistencia al ser acelerados.
En la terminología de Newton movimiento significa fuerza
cantidad de movimiento que actualmente se define por el masa inercial 
producto de la masa (m) y la velocidad (v) de la partícula:
aceleracio n
3) El término de "fuerza motriz" que empleó Newton
p = mv (3.4)
para enunciar su segunda ley se refiere a una fuerza
La variación de la cantidad de movimiento con el tiempo es resultante o fuerza no equilibrada que al actuar sobre un
su derivada respecto al tiempo; por tanto la segunda ley de cuerpo le comunica una aceleración.
Newton expresa que:
En general el cuerpo está sometido a la acción simultánea de
dp d(mv) un número cualquiera de fuerzas, cuya resultante o fuerza neta
F  (3.5) diferente de cero, produce la aceleración. Matemáticamente
dt dt
se expresa:
Si la masa es independiente del estado de movimiento
encontramos que:  Fj = ma (3.7)

d(mv) dv ó expresado equivalentemente en términos de sus


m  ma componentes rectangulares:
dt dt
Fjx = max; Fjy = may; Fjz = maz (3.8)
F= ma
Es importante notar que la segunda ley de Newton contiene 3
ideas fundamentales: 3.5 TERCERA LEY DE NEWTON:
ACCION Y REACCION
1) En todos los casos, la dirección de la aceleración es la
misma que la de la fuerza. Las fuerzas en la naturaleza se presentan siempre por pares:
Esto es cierto bien se encuentre el cuerpo inicialmente en "Para cada acción hay una reacción de
reposo o bien moviéndose en cualquier dirección y con igual módulo pero de sentido opuesto"
cualquier velocidad; por ejemplo para retardar el movimiento
de un cuerpo es necesario ejercer una fuerza en sentido
opuesto a su velocidad y por tanto el movimiento resultará Por ejemplo, si el cuerpo A ejerce una fuerza FAB (acción)
con aceleración negativa o sea opuesta a la velocidad. sobre el cuerpo B, simultáneamente se ejerce FBA (reacción)
de B sobre A, véase la figura 3.5(b) de tal manera que ambas
2) Para un cuerpo dado, la razón del módulo de la fuerzas están en la misma dirección pero con sentidos
fuerza al de la aceleración es siempre la misma, o sea es opuestos y cumplen la relación:
constante.
FBA = – FAB (3.9)
F/a = constante ( para un cuerpo dado)
N(reacción)
En efecto la experiencia demuestra que si duplicamos la
fuerza aplicada sobre un cuerpo su aceleración también se
duplica, verificándose que: FBA FAB
A B
F' 2F F
 
a ' 2a a P(acción)

Esta razón constante entre la fuerza y la aceleración puede (a) (b)


considerarse una propiedad del cuerpo denominada su masa
(m), de donde: Figura 3.6 Interacción mutua
F
m (3.6)
a
En la figura 3.6 (a) el bloque presiona contra el piso con una
Este resultado tiene un profundo significado pues fuerza P (acción), mientras que el piso sostiene al bloque con
observemos que en esta definición de masa no se tiene en una fuerza N (reacción). Debe tenerse presente que estas
cuenta en absoluto la constitución del material o estructura fuerzas se aplican sobre cuerpos diferentes, llamándose a
del cuerpo. Además recordemos que la fuerza es la medida cualquiera de ellas acción y a la otra reacción. Por ejemplo en
de una interacción entre dos cuerpos, que bien puede ser una la figura 3.6 (b). Si A es la Tierra y B es la Luna; la Tierra
acción a distancia y la aceleración es un concepto netamente atrae a la Luna con una fuerza FBA (acción) pero a su vez la
cinemático. Luna atrae a la Tierra con otra fuerza FAB de igual valor pero
de sentido opuesto y cada una de las fuerzas tiene su punto de
La masa de un cuerpo es un escalar numéricamente igual a la aplicación en cuerpos diferentes: acción sobre la Luna y
fuerza necesaria para comunicarle la unidad de aceleración reacción sobre la Tierra.
(fuerza por unidad de aceleración)
Cuando es necesaria una gran fuerza para variar la velocidad
de un cuerpo, la masa del cuerpo es grande y decimos Otro ejemplo es la de una persona que estando sentada en
entonces que el cuerpo tiene gran inercia. Si sólo es una silla de ruedas ejerce una fuerza sobre un muro de
necesaria una pequeña fuerza por unidad de aceleración se concreto, este no se mueve a pesar de que una fuerza actúa
Daniel Fernández Palma / Raúl Zavala Sánchez Dinámica 71

sobre él, pero la reacción del muro que es una fuerza igual y fuerza por unidad de masa ejercida por la Tierra sobre
de sentido contrario, actúa sobre la persona y hace que la silla cualquier objeto. Es igual a la aceleración de la gravedad
se mueva hacia atrás.
Cuando estamos de pie sobre una balanza de muelles,
nuestros pies aprecian la fuerza ejercida sobre nosotros por la
balanza para equilibrar nuestro peso. La fuerza que equilibra
3.6 UNIDADES DE FUERZA MASA Y nuestro peso se denomina peso aparente. Si no existiese
ACELERACION ninguna fuerza para equilibrar nuestro peso, como sucede en
la caída libre, el peso aparente seria cero. Esta condición
En el sistema internacional o en el sistema M.K.S la unidad denominada ingravidez es la que experimentan los
de masa es el kilogramo y la unidad de fuerza es el newton astronautas, en los satélites que giran alrededor de la Tierra o
definido como la fuerza que aplicada al kilogramo patrón le dentro de un ascensor en caída libre con aceleración g
comunica la aceleración de 1 m/s2. El peso de un cuerpo también varía en los diversos lugares de
1 NEWTON (N) = 1 kg. m/s2 la Tierra siendo mayor en los polos y mínimo en el ecuador,
en tanto que la masa permanece constante.
En el sistema métrico cgs, la unidad de masa es el gramo
(10–3 kg), y la de distancia es el centímetro (10–2 m). La m = W/g (3.11)
unidad de fuerza correspondiente se llama dina EJEMPLO 3.3 Una persona utilizando una balanza determina
1 dina = 1g. cm/s2 su masa que es igual a 80 kg. ¿Cuál es su peso?.

En el sistema británico la unidad de fuerza es la libra (o libra- Solución: Con la ecuación 3.10 hallamos el peso del cuerpo
fuerza) y la de masa es el slug. en SI

La unidad de aceleración es el pie/s2 W = mg = (80 kg)(9.8 m/s2) = 784 kg. m/s2 W = 784 N

1 libra = 1 slug.pie/s2 NOTA IMPORTANTE En la vida diaria usamos


incorrectamente las unidades del sistema internacional. Con
Un cuerpo de 1 kg de masa tiene un peso aproximadamente frecuencia se dice por ejemplo que “este libro pesa 2 kg”, lo
de 2,2 lb en la superficie terrestre. que queremos decir es que la masa del libro (quizá
determinada indirectamente pesándola) es de 2 kg. Tenga
cuidado y evite este error, el peso (una fuerza) se mide en
Tabla 3.1 newtons, y la masa en kilogramos
UNIDADES DE FUERZA MASA Y ACELERACION
Sistema Fuerza Masa aceleración EJEMPLO 3.4 Un muchacho de 560 N que viaja en la
SI Newton ( N) Kg m/s2 dirección +x se detiene abruptamente, la componente x de la
cgs Dina (Dina) G cm/s2 fuerza neta que actúa sobre él es –1.5104 N. ¿Cuál es su
Británico Libra (Lib) Slug pie/s2 aceleración?
Solución: Nótese que el peso es P = 560 N y por tanto la
Equivalencias:
masa es:
1 Newton (N)..….................... = 10 5 dinas
1 libra (lb).………………..... = 4,448 N P 560N
m= = = 57,1 kg
1 slug (slug)......................….. = 14,56 kg g 9,8 m / s 2
1 pie (ft)............= 12 pulgadas = 0,3048 m
1 pulgada (in).......................... = 2,54 cm Luego la aceleración experimentada es

F  1,5  10 4 N
a= = = – 262,5 m/s2
3.7 MASA Y PESO m 57,1kg
La fuerza más común en nuestra experiencia diaria es la
fuerza de la gravedad o atracción que ejerce la Tierra sobre Esta aceleración también puede escribirse como a = – 26,8g,
todos los cuerpos que están sobre ella. Esta fuerza se llama Es decir 26,8 veces la aceleración de la gravedad
peso del cuerpo. Cuando un cuerpo es abandonado y se deja
caer libremente, la única fuerza que actúa sobre él es su peso
W = mg y su aceleración es la de cualquier cuerpo que cae 3.8 MARCOS DE REFERENCIA NO
libremente, es decir la aceleración de la gravedad cuyo valor INERCIALES
promedio es g = 9.8 m/s2 = 980 cm/s2 = 32,2 pie/s2.. De
acuerdo con la segunda ley de Newton, la expresión de la Consideremos un ascensor en cuyo interior se encuentra un
fuerza peso es: observador tratando de medir su peso con una balanza de
resorte y encuentra que su peso es lo normal cuando el
W = mg (3.10) ascensor sube o baja a velocidad constante. pero resulta
donde m es la masa del cuerpo; mayor cuando el ascensor acelera hacia arriba y menor
cuando lo hace hacia abajo
El vector W actúa verticalmente dirigido hacia el centro de la
Tierra. Como g es idéntico para todos los cuerpos en un El ascensor en movimiento rectilíneo uniforme (velocidad
punto determinado, llegamos a la conclusión de que el peso constante) es un sistema inercial y de acuerdo con el
de un cuerpo es proporcional a su masa. El vector g de la principio de inercia el reposo es equivalente al movimiento
ecuación (3.10) se denomina campo gravitatorio y es la rectilíneo uniforme por tanto el peso del hombre en el interior
del ascensor no difiere de su peso normal. En cambio cuando
72 Dinámica Daniel Fernández Palma / Raúl Zavala Sánchez

el ascensor acelera deja de ser un sistema inercial o sea se ómnibus al tomar una curva da un viraje hacia la izquierda y
convierte en un sistema no inercial los pasajeros son impulsados hacia la derecha apretándose
contra la ventanilla lateral exactamente como si una fuerza
atractiva surgiera de pronto entre pasajero y ventanilla.
Si en la ecuación (3.12), hacemos dv/dt = ; encontramos
a = a' + 
multiplicando por la masa se encuentra que
F = F' + Fc (3.14)
este resultado es completamente distinto de lo hallado en el
caso inercial y se nota que ahora tenemos una fuerza
adicional y extraña (Fc) y por tanto F  F'
Fig 3.7. Ascensor en movimiento Cualquier “fuerza” que pueda ver un observador en
aceleración, es una fuerza ficticia que también se llama
pseudofuerza, o fuerza no inercial. Este tipo de fuerza
Supóngase dos observadores, uno en tierra (A) y el otro (B) extraño, es bastante distinto al de las fuerzas que hemos
dentro del ascensor; están en diferentes marcos de referencia encontrado hasta ahora, que se deben a influencias externas.
inerciales, moviéndose entre sí a una velocidad constante v Las fuerzas reales tienen un origen o agente identificable, y
(velocidad del ascensor) Estos observadores miden que un no se deben al movimiento del observador.
cuerpo en movimiento tiene distintas velocidades u y u' (ver
el capítulo de cinemática, ecuación 2.70) Una fuerza ficticia muy común es la fuerza centrífuga, que se
observa en sistemas con movimiento circular uniforme o
u = u' + v variado
Veamos como los observadores A y B miden la aceleración.
Esto es, en la ecuación anterior hallamos sus derivadas
respecto al tiempo: EJEMPLO 3.5 Un péndulo simple se encuentra dentro de un
vehículo en movimiento con aceleración a = 5 m/s2. ¿Qué
du du' dv ángulo forma el péndulo con la vertical?
  (3.12)
dt dt dt
Solución: La figura muestra la posición de equilibrio del
Estando los observadores en marcos de referencia inerciales péndulo dentro del vehículo en marcha
v es constante y su derivada igual a cero. Entonces:

θ
du du' a
 ó a = a'
dt dt
Como la masa del cuerpo que se mueve no depende de la
velocidad del observador obtenemos:
ma = ma'
o bien según la segunda ley de Newton: Fig.3.8 Posición de un péndulo en
un sistema acelerado
F = F' (3.13)
Por tanto las fuerzas medidas en ambos sistemas son iguales. Un observador en el interior del vehículo informará que la
Esto es: masa pendular m permanece en equilibrio bajo la acción de
las tres siguientes fuerzas: peso mg , fuerza ficticia ma y la
Los observadores en diferentes marcos de referencia tensión del hilo T
inerciales concuerdan en la fuerza neta que actúa sobre
cada cuerpo
Como demostraremos en el capitulo X de FISICA Vol III 
(sección 10.4), en relatividad especial la relación entre las
velocidades es más complicada que la ecuación (2.70), sin T
embargo, es notable que aún en dichos casos, sigue siendo
ma
cierto que dos observadores, en marcos de referencia inercial
distintos, no pueden decir por medio de un experimento, cual
de ellos se mueve y cual está en reposo 
mg
Supongamos ahora que v no es velocidad constante. Por tanto
dv/dt no es cero. Uno o ambos, se encuentran en marcos no
inerciales de referencia. Según la ecuación (3,12), el
observador B medirá que un objeto tiene una aceleración Figura 3.9 diagrama del cuerpo libre
distinta de la que mide el observador A, y los dos no se de la masa pendular
pondrán de acuerdo acerca de las fuerzas que actúan, esto
explica porqué el peso de un cuerpo es diferente de lo normal En la figura 3.9 la tangente del ángulo está dada por:
en un ascensor acelerado o como ocurre en el interior de un
Daniel Fernández Palma / Raúl Zavala Sánchez Dinámica 73

ma 5 EJEMPLO 3.7 Un bloque de masa m1 = 40 kg que está sobre


tan = = una superficie horizontal sin fricción, está unido mediante
mg 9,8 una cuerda que pasa por una polea a otro bloque suspendido
 = arctan(0,51) = 27° de masa m2 = 10 kg. Si la masa de la cuerda es despreciable y
la polea es totalmente lisa, calcular la aceleración de los
bloques y la tensión en la cuerda.
3.9 ALGUNAS APLICACIONES DE
LAS LEYES DE NEWTON N T
m1
Las tres leyes de Newton son suficientes para el estudio de m2
m1 T
cualquier problema de mecánica, con auxilio de algunas
definiciones complementarias.
m2 m1g
m2g
EJEMPLO 3.6 La máquina de ATWOOD es un dispositivo DCL(m1) DCL(m2)
que se empleó en el siglo XVIII para medir la aceleración de
la gravedad. Se compone de dos masas m1, m2 unidas por una Figura 3.11 la masa total m1+m2 es accionada tan solo por el
cuerda de peso despreciable que pasa por una polea ligera peso de m2
con una fricción despreciable figura.3.9 Si m1 = m2, el
sistema permanece en reposo en cualquier posición Solución: Para hacer el diagrama del cuerpo libre (DCL), es
equilibrándose sus pesos entre sí. Si m2 > m1; m2 se acelera necesario identificar todas las fuerzas que actúan sobre cada
hacia abajo y m1 hacia arriba. uno de los bloques.

La aceleración es constante. Si m2 es ligeramente mayor que Sobre el bloque de masa m1: .


m1, la aceleración es pequeña y su valor se puede medir con - su peso ….…….……….W1 = m1g
mucha facilidad a partir del cual se calcula g. - tensión en la cuerda……..T = ¿
- la reacción del piso……..N = ¿
Para escribir la ecuación dinámica elegimos el eje X,
horizontal y eje Y, vertical. En este sistema la aceleración “a”
T de m1 sigue la dirección de T
 Fjx : T = m1a (1)
T
m1 m1 Y desde que m1 no tiene movimiento vertical
m1g m2  Fjy : N – W1 = 0 ó N = W1 = m1g
m2
Para el bloque de masa m2:
m2g
- su peso …......……… W2 = m2g
- tensión en la cuerda . T = ¿
Figura 3.10 la diferencia de los pesos, acciona a la suma de
masas . La masa m2 sólo tiene aceleración vertical descendente y su
valor es también “a” (cuerda inextensible). Luego aplicando
la segunda ley de Newton tenemos:
En la actualidad, conociendo el valor de g; la máquina de  Fiy : T – m2g = – m2a (2)
Atwood puede servirnos para encontrar las aceleraciones de
los cuerpos suspendidos y luego sus velocidades y Eliminando T entre (1) y (2) obtenemos:
posiciones.
m1a – m2g = – m2a
La tensión en la cuerda es la misma a uno y otro lado de la
de donde:
polea. Para aplicar la segunda ley de Newton a este sistema
supondremos que la polea gira en sentido horario y por tanto m2g 10g
m1 asciende y m2 desciende ambos con la aceleración de a   0,2g
m1  m 2 40  10
valor “a” . Las ecuaciones son:
T – m1g = m1a (1) a = 1,96 m/s2
T – m2g =- m2a (2) De la ecuación (1): T = m1a = 40(0,2g)
de (1) y (2) tenemos: T = 78,4 N

(m 2  m1 )g
a
m1  m 2 3.10. FUERZAS DE FRICCION
Fuerza de fricción, es la fuerza que aparece en la superficie
En (1) reemplazando el valor de “a” y despejando para T se
de contacto entre dos cuerpos oponiéndose al movimiento
tiene:
relativo de estos (Fig 3.12). La fricción o rozamiento se debe
2m1m 2 g a las fuerzas intermoleculares en la superficie libre de los
T cuerpos; interviene la adhesión y cohesión
m1  m 2
74 Dinámica Daniel Fernández Palma / Raúl Zavala Sánchez

F
La experiencia nos indica que es más fácil mantener el
f movimiento de un cuerpo sobre otro que iniciarlo, por esto,
en general k < s y consecuentemente fk < fs. En un plano
inclinado, cuando el movimiento es inminente por acción de
una componente de su peso para un ángulo s tenemos que el
Figura 3.12 Si F > f se producirá aceleración cuerpo está en equilibrio; figura.3.16 ( = s , k ).
En realidad, observando las superficies en contacto
microscópicamente, no descansa por completo la una sobre la N
otra sino en las partes prominentes solamente (Fig.3.13) ésta, fk
es la razón por la que la fuerza de fricción no dependa del
área en contacto.
mg.sen 
 mg cos 
mg

Figura 3.16 el ángulo  es menor para mantener


Figura.3.13 fuerzas electromagnéticas entre el movimiento que para iniciarlo
superficies en contacto
Entonces tenemos: Wsens – fs = 0
FUERZA DE ROZAMIENTO ESTATICO: N – mgcoss = 0
De donde: mg sens = fs
Si gradualmente intentamos el movimiento de un cuerpo mg coss = N
sobre otro, mientras el cuerpo está en reposo, la fuerza de
rozamiento va creciendo desde cero hasta un valor máximo fs Dividiendo miembro a miembro:
en que el movimiento es inminente. (figura 3.14) Esta es la
fuerza de rozamiento estático y es proporcional a la fuerza fs s N
tgs = 
normal, esto es: N N
fs = sN (3.15) Luego la relación entre el coeficiente de fricción estática y el
ángulo de inclinación del plano inclinado está dada por:
Donde: N es la fuerza normal o fuerza de contacto que
mantiene unidas a las superficies y s es el coeficiente de s = tgs (3.17)
rozamiento estático.
Si se inicia el movimiento es necesario disminuir un tanto la
inclinación del plano inclinado para mantener el
N
movimiento a velocidad constante (a = 0). En tal caso las
F ecuaciones conducen al siguiente resultado:
fs v
 k = tg  k (3.18)
mg ( k <  s )

Figura 3.14 el movimiento es inminente si fs = F


EJEMPLO 3.8. Encontrar la aceleración de la masa m = 10
kg si el coeficiente de fricción cinético es 0,2 y la fuerza es
FUERZA DE ROZAMIENTO CINETICO: constante como se indica en la figura 3.17(a), determine
también la fuerza de contacto con el piso y la distancia que
Estando el cuerpo en movimiento siempre existe la fuerza de recorre en un tiempo de 10 segundos si vo = 0.
rozamiento, La misma que se llama fuerza de rozamiento
cinético.
F=30N N F
fk = kN (3.16)
37° 37°
donde: N es la fuerza normal y k es el coeficiente de
rozamiento cinético. m fk

N
(a) (b) W =mg
F v
fk
Figura 3.17 la normal no siempre es igual al peso

mg En el diagrama del cuerpo libre se muestra todas las fuerzas


que actúan sobre el bloque en movimiento. las ecuaciones
escalares del movimiento son:
Figura 3.15 con F’ = f k el movimiento es rectilíneo uniforme
F cos37° – fk = ma (1)
Daniel Fernández Palma / Raúl Zavala Sánchez Dinámica 75

Fsen37° + N – mg = 0 (2) encontró que la aceleración normal o centrípeta está dada por
la ecuación (2.63)
de la definición de fuerza de fricción:
fk = kN (3) v2
an = (3.19)
R
de la ecuación (2) despejamos el valor de la fuerza de
contacto (N). De acuerdo con la segunda ley de Newton (F = ma), la fuerza
F que se necesita para mantener en movimiento circular
N = mg – Fsen37° (4)
uniforme a una masa m es:
reemplazando datos: m = 10 kg, g = 9,8 m/s2,
v2
F = 30N. obtenemos: F= –m ur (3.20)
R
N = 80 Newton.
donde el vector unitario ur apunta hacia afuera partiendo del
En la ecuación (3) reemplazamos k = 0,2 y N = 80 N, centro del movimiento circular y el signo menos indica que la
obtenemos: fuerza es hacia el centro del círculo
f k = 16,0 N Las fuerzas que apuntan hacia el centro del círculo se llaman
fuerzas centrípetas. En el caso del vaso moviéndose en
De (1) la aceleración de la masa es:
círculo, la tensión en la cuerda es la fuerza que mantiene el
movimiento y dado que la cuerda va hacia el centro del
F cos37o  f k 30(4 / 5)  16,0
a  círculo, la dirección de la fuerza de tensión sobre el vaso
m 10 apunta al centro de la circunferencia.
a = 0,8 m/s2
La distancia recorrida en el movimiento uniformemente EJEMPLO 3.9 Si la Luna cuya masa es de 7,341022 kg se
acelerado cuando la velocidad de partida es cero esta dado mueve en trayectoria circular en torno a la Tierra con un
por: periodo de 27 d, 7 h, 43 min y un radio orbital de 3,84108
x = ½ at2 m. Calcular la frecuencia angular y la fuerza centrípeta sobre
la Luna.
reemplazando la aceleración y t = 10 s se obtiene:
x = ½ (0,8 m/s2)(10 s)2 = 40 m
Solución Expresando el periodo de la Luna en segundos se
tiene T = 2,36106 s. Por tanto la frecuencia angular esta
3.11 FUERZAS Y MOVIMIENTO CIRCULAR dado por:
2 6,28
Si hacemos girar con suficiente rapidez en trayectoria = = = 2,6610-6 rad/s
circular, un vaso conteniendo agua, se observa que el agua no T 2,36  106
se derrama aun cuando la trayectoria se encuentre en el plano Recordando que la velocidad tangencial v y la frecuencia
vertical. angular  están relacionados por v = R; el valor de la
fuerza centrípeta está dado por:

v2
F=m = 2mR
R
reemplazando en la fórmula anterior los datos disponibles
obtenemos:
F = (2,6610-6)2 (7,341022)(3,84108)
F = 1,991020 N
Para magnitudes a escala humana, ésta es una fuerza
Figura 3.18 El contenido del balde no se vierte descomunal, si tuviésemos que reemplazar esta fuerza con la
tensión de un cable de acero, el diámetro de este cable sería
del orden de 1010 m (mucho mayor que el diámetro terrestre)
En este movimiento tal como se ilustra en la figura (3.18) la
cuerda de sujeción se encuentra tensa. Lo que siente la
mano del experimentador es la tensión en la cuerda. Para EJEMPLO 3.10 Una estación espacial tiene la forma de una
medir esta tensión bastaría insertar en la cuerda un resorte; el doble rueda (dona). Cada rueda es un tubo circular de gran
estiramiento del resorte es la medida de la tensión. Una longitud, en cuyo interior se mueven los astronautas
cuerda en tensión tira de aquello de lo cual está conectada; en Al girar la estación espacial en torno a su eje, los astronautas
este caso, de la mano en un extremo y del vaso en el otro. experimentan la acción de la fuerza centrifuga (fuerza
Sería la única fuerza horizontal sobre el objeto. Esta fuerza ficticia) que los impulsa al lado más alejado del centro
neta horizontal aceleraría al vaso en el plano horizontal. Esto dándoles la sensación de peso. ¿Cual deberá ser la velocidad
resulta concordante con lo que se encontró en el estudio tangencial del borde de la estación espacial para que el
del movimiento circular uniforme, en el cual, aun cuando la “peso” del astronauta sea igual al que tiene en la Tierra.
rapidez es constante, existe una aceleración. Hay aceleración Suponga que el radio exterior de la estación es de 40 m.
porque el vector velocidad está cambiando de dirección. Se
76 Dinámica Daniel Fernández Palma / Raúl Zavala Sánchez

La ley de fuerza experimental, para la fuerza de resistencia


depende de modo muy complicado de la forma y del tamaño
del cuerpo que se mueve. A bajas velocidades depende
linealmente de la velocidad y está relacionada con la
viscosidad del medio, pero a altas velocidades se establecen
efectos relacionados con la turbulencia del medio y la fuerza
de resistencia es proporcional al cuadrado de la velocidad.
Para un fluido dado, la fuerza de resistencia tiene la siguiente
forma:
FD = av + bv2 ( 3.21)
los coeficientes a y b dependen de la forma del objeto y del
medio en el cual se mueve. El primer término domina a bajas
velocidades y en tales casos la ecuación anterior se aproxima
a:
Figura 3.19 Estación espacial diseñado para generar
gravedad artificial FD = av (3.22)
En muchos casos, como el movimiento de un automóvil en la
carretera, o el de un paracaidista en el aire, domina el
segundo término. De acuerdo a experimentos, el segundo
término tiene la forma:
FD = ½ ACDv2 (3.23)
En la cual  es la densidad del medio, A es el área máxima
de la sección transversal que presenta el objeto,
perpendicularmente a su desplazamiento y CD es el
coeficiente de resistencia; CD es adimensional y depende de
R la forma del cuerpo por ello se denomina factor de forma.
Cuando un objeto se desplaza dentro de un fluido, la fuerza
de resistencia aumenta hasta equilibrar a la fuerza impulsora
del movimiento, la velocidad para la cual ocurre este
equilibrio se denomina velocidad límite o terminal. Por
Figura 3.19 La fuerza centrífuga hace que los astronautas ejemplo de la ecuación (3.23) vemos que si la fuerza de
tengan peso y puedan caminar resistencia es tal que logra equilibrar el peso en la caída de un
cuerpo en la atmósfera obtenemos
FD = mg = ½ ACDv2
Solución. La fuerza centrífuga en valor igual a la fuerza
centrípeta está dado por: la velocidad terminal en este caso es:
2
v 2mg
Fc = m v (3.24)
R AC D
reemplazando la fuerza centrífuga por el peso mg y EJEMPLO 3.11 En un fluido como el agua o el aceite, los
despejando la velocidad hallamos: coeficientes a y b de la Ec(3.21) son a = 0,2 kg/s, b = 0,1
kg/m. ¿A qué velocidad los dos términos de dicha ecuación
v= gR = (9,8)(40) = 19,8 m/s son iguales?.
Esta velocidad es bastante alta; representa algo más de 70 Solución Si designamos con vL la velocidad a la cual se
km/h y deberá constituir uno de los factores más importantes igualan los dos términos de la ecuación (3.21) tenemos:
en el diseño de dicha estación sobre todo en lo que concierne
a la resistencia de los materiales. avL = b vL2 ó vL = a/b = 0,2/0,1
vL = 2 m/s

3.11 FUERZAS DE RESISTENCIA EJEMPLO 3.11 Para una esferilla de acero de 5 g de masa
que cae dentro del agua calcular el valor de la velocidad para
Un cuerpo que cae en el seno de un líquido o un paracaidista la cual la fuerza de resistencia equilibra al peso:
que cae desde una gran
Solución. Como la caída es en agua la situación corresponde
altura, están sujetos a una fuerza de resistencia, que a bajas velocidades por consiguiente:
denominamos, fuerza de retardo, o resistencia viscosa. En el
caso de un cuerpo que se desliza sobre una superficie, la mg = a vL o vL = mg/a = 0,0059,8/0,2
fuerza de fricción depende de la normal. vL = 0,2 m/s
En el caso de los fluidos no hay una fuerza normal, pero
sigue manifestándose una fuerza de resistencia que trata de
impedir el movimiento. Estas fuerzas de resistencia así como 3.12 FUERZAS FUNDAMENTALES
la fricción, actúan en sentido opuesto al movimiento; sin
En las secciones anteriores hemos tratado con una gran
embargo la fuerza de resistencia de los fluidos depende de la
variedad de fuerzas tales como, fuerza normal, fricción,
velocidad del objeto a través del medio.
Daniel Fernández Palma / Raúl Zavala Sánchez Dinámica 77

tensión, fuerza elástica del resorte etc. Entre las diversas limitada, que las fuerzas electrodébil y gravitacional se
fuerzas que manejamos, las fundamentales desempeñan un pueden apreciar. La fuerza gravitacional solo domina a escala
papel privilegiado. Son las fuerzas de las cuales se derivan en astronómica. Esto es porque la materia, está dispuesta en
último término, las fuerzas secundarias. Por ejemplo la fuerza combinación eléctricamente neutra, en la cual las fuerzas
peso o fuerza debido a la gravedad, es tan sólo un caso electromagnéticas se reducen a cero
especial de la fuerza fundamental, la gravitación universal,
que es la que describe la fuerza entre objetos cercanos o
lejanos. Esta fuerza la comprendió por primera vez Newton. 3.13 PROBLEMAS RESUELTOS
La fuerza que ejerce una espiral rígida de un alambre, el
resorte, depende de las fuerzas eléctricas fundamentales Es
fácil deducir la relación entre gravedad y gravitación Segunda y Tercera ley de Newton
universal. Sin embargo sería casi imposible calcular la
fuerza del resorte a partir de las fuerzas fundamentales
1. Se empuja con fuerza constante un cuerpo en línea recta
eléctricas entre sus átomos constituyentes, pero como es fácil
sobre una superficie horizontal y sin rozamiento. El aumento
determinar la fuerza del resorte, mediante experimentos, no
de su velocidad en un intervalo de 10 s es de 5 km/h.
nos interesa mucho calcular la fuerza del resorte a partir de
Cuando se aplica además una segunda fuerza constante en la
las fuerzas fundamentales.
misma dirección, la velocidad aumenta a 15 km/h en un
La comprensión de lo que son las fuerzas fundamentales es intervalo de 10 s. ¿Cómo son en comparación ambas fuerzas?
relativamente reciente. En el desarrollo histórico de las
Solución. Las aceleraciones producidas por las fuerzas son
fuerzas fundamentales se han producido varias unificaciones.
respectivamente:
Así, las que antes se consideraban como fuerzas claramente
distintas y fundamentales, se han vuelto tan sólo aspectos de v 5 km / h
una misma fuerza fundamental. Así por ejemplo, las fuerzas a1 = = = 0,5 km/h por segundo
eléctricas y las magnéticas constituyen aspectos de la fuerza t 10 s
electromagnética v 15 km / h  5 km / h
a2 = = = 1 km/h por seg
Ahora suponemos que en la naturaleza sólo existen tres t 10 s
fuerzas fundamentales:
Se puede ver que a2 = 2a1; por tanto, la segunda fuerza es dos
1. la gravitación universal, veces la primera
2. la fuerza electrodébil
3. la fuerza nuclear o fuerte

Uno de los mayores descubrimientos de Newton fué la 2. Una fuerza Fo causa en un cuerpo una aceleración de 6x106
ecuación de la fuerza de gravitación universal.. Einstein en su m/s2. Otra fuerza causa en el mismo cuerpo una aceleración
ley de la relatividad general, refinó esa ecuación haciendo de 9106 m/s2. Hallar a) la magnitud de la segunda fuerza b)
algunas nuevas predicciones de pequeños efectos no la aceleración del objeto si las dos fuerzas actúan
predichos por la gravitación newtoniana. Así mismo simultáneamente sobre el objeto 1) en la misma dirección 2)
Einstein realizó esfuerzos persistentes, pero sin éxito, para en direcciones opuestas 3) si las dos fuerzas son
unificar la gravitación con las demás fuerzas fundamentales. perpendiculares entre sí.
La fuerza electrodébil es un descubrimiento reciente que
parte de la teoría propuesta por Sheldon L. Glashow, Abdus Solución a) Desde que las aceleraciones producidas son
Salam y Steven Weinberg a mediados de la década del 70 y directamente proporcionales a las fuerzas aplicadas tenemos:
comprobada experimentalmente a principios de 1980. Se
Fo 6  106 2 3
otorgaron dos premios Nóbel por este trabajo, que tuvo éxito  = ó F1 = 2 Fo
al unificar dos fuerzas que antes se pensaba eran F1 9  106 3
independientes y fundamentales: la fuerza débil responsable
b-1) si las dos fuerzas actúan en la misma dirección, la fuerza
principal de los procesos radiactivos raros, e importante en la
3 5
evolución del universo y las fuerzas del electromagnetismo resultante sobre el cuerpo es F = Fo + 2
Fo = 2
Fo. Puesto
A partir de la década de 1980, se han renovado los esfuerzos que las aceleraciones guardan la misma relación que las
para llegar donde Einstein no pudo, y llevar a la gravitación fuerzas, la aceleración producida es
al programa de unificación. Trabaja aún en este tema
fascinante el incomparable científico de estos tiempos a= 5
ao = 5
(6x106) = 15x106 m/s2
Stephen W. Hawking. 2 2

En la escala de la vida cotidiana, de las fuerzas secundarias b-2) Si actúan en direcciones opuestas, la fuerza resultante es
5
que se han descrito en este capítulo sólo aparece la gravitación F= 2
Fo – Fo = ½ Fo y la aceleración producida es:
universal en la forma de gravedad. Todas las demás fuerzas
secundarias se deben a la fuerza electromagnética, que a = ½ ao = ½(6×106) = 3×106 m/s2
agrupa a átomos y moléculas que forman la materia
ordinaria. No vemos directamente la fuerza nuclear en b-3) Si las fuerzas son perpendiculares, el módulo de la
nuestra escala; sujeta los núcleos atómicos de modo tan resultante es:
apretado, que para fines prácticos podemos imaginar que
estos son trozos indivisibles de materia. Sólo cuando F= (Fo )2  ( 32 Fo )2 = 13 F
4 o
describamos la energía nuclear o la composición de las
estrellas, entrarán en juego las fuerzas nucleares La aceleración respectiva es:
Las fuerzas fundamentales se ordenan según su intensidad 13 13
decreciente en: nuclear, electrodébil y gravitacional. Sólo es a= 4
ao = 4
(6106) = 10,8106 m/s2
debido a que la fuerza nuclear actúa a una distancia tan
78 Dinámica Daniel Fernández Palma / Raúl Zavala Sánchez

3. a)Un cuerpo está sometido a la acción de dos fuerzas F1 y F = 4t – 5


F2 como se indica en la figura. Determinar la aceleración a
del objeto b) una tercera fuerza F3 de modo que el cuerpo Luego calculamos la aceleración:
está en equilibrio estático. Determinar F3 F 4t  5
a  = 0,4t – 0,5
m 10
F1 = 20N y reemplazando t = 1,5 s hallamos el valor de la aceleración a
= 0,1 m/s2
10 kg x (b) resolviendo la ecuación p = 0
30°
2t2 – 5t + 2 = 0
F2=30N (t – 2)(2t – 1) = 0
La solución no descartada es t = 0,5 s y la fuerza en el
Figura 3.20 Buscando la fuerza resultante
instante t = 0,5 s, es:
F = 4 (0,5) – 5 = – 3 N
Solución La componente X de la resultante es F x = 0 +
F=–3N
F2cos30° = 0 + 30cos30 = 26 N
La componente Y es Fy = F1 + F2sen30° = 20 –30sen30 = 5 N
6. Una caja se desliza hacia abajo por un plano inclinado.
Dividiendo entre la masa hallamos las componentes de la
Dibujar un diagrama que muestre, las fuerzas que actúan
aceleración:
sobre ella. Para cada una de las fuerzas del diagrama indicar
ax = Fx/m = 26/10 = 2,6 m/s2 la fuerza de reacción.
ay = Fy/m = 5/10 = 0,5 m/s2
Solución En el diagrama se muestra los siguientes vectores
así tenemos el vector aceleración y su módulo fuerza: El peso W y su reacción W'; La fuerza de contacto
entre caja y plano inclinado W1 y su reacción W1'
a = (2.6 , 0.5) a = 2,65 m/s2
su dirección respecto al eje x es: W1
 = arctan(Fy/Fx) = arctan(0,5/2,6) = 11°
La fuerza equilibrante F3 debe ser opuesta a la fuerza
resultante de F1 y F2. Esto es: W'
W W1'
F3 = – 26 i – 5 j

4. Una sóla fuerza de 10 N actúa sobre una partícula de Figura 3.21 Acción y reacción
masa m La partícula parte del reposo y se mueve sobre una
7 Un bloque de hielo liberado del reposo en la parte superior
recta a lo largo de una distancia de 18 m en 6 s. Hallar su
de una rampa de 4,00 m, llega a la base en 1,60 s ¿qué ángulo
masa m.
forma la rampa con la horizontal?
Solución En el movimiento uniformemente variado la
aceleración está dada por:
Solución La fuerza neta en la dirección del movimiento
2x 2(18) (suponiendo nula la fricción) es: F = mg.sen. Por
a= = = 1 m/s2
t2 62 consiguiente la aceleración del móvil es: a = F/m = g.sen

Luego la masa de la partícula es:


F 10 v
m= = = 10 kg L h
a 1

5. El pistón de un motor de explosión pesa 10 kg y está
sometido a la acción de fuerzas variables de tal modo que su
cantidad de movimiento lineal está dado por la siguiente Figura 3.22 Fuerza neta a lo largo del plano inclinado
ecuación: p = 2t2 – 5t + 2; 0 < t < 2 donde p queda
expresado en kg.m/s cuando t se mide en segundos.
Determinar: a) la aceleración del pistón cuando t = 1,5 s. b) el Como la distancia recorrida es L, aplicamos la fórmula de
valor de la fuerza en el instante en que se anula la cantidad distancia para el MRUV
de movimiento lineal.
L = ½ at2 = ½(g.sen)t2
Solución: (a) calculamos la fuerza: aplicando su fórmula de
2L 2(4)
definición de donde sen  = = = 0,3188
gt 2 9,8(1,6)2
dp d
F  (2t 2  5t  2) = 4t – 5
dt dt  = arcsen(0,3188) = 18,6°
Daniel Fernández Palma / Raúl Zavala Sánchez Dinámica 79

8. La trayectoria que origina una fuerza al actuar sobre un F = F1(cos37° – µsen37°) – µmg
cuerpo de 2 kg de masa, está determinada por las ecuaciones
:x = 6t2 – 5t; y = 4t2 + 3; z = 3t4 – 3t + 1, donde x,y,z se reemplazando valores obtenemos la fuerza resultante:
mide en metros y t en segundos. Calcular el módulo de la F = 2 + 17t2 (3)
fuerza cuando t = 1/3 s.
El impulso, está dado por la siguiente expresión integral:
Solución. Calculando las componentes de la aceleración
17 3 3
 Fdt = 0 (2  17t
3 2
dx dv x I= )dt = [2t + 3
t ]
x= 6t2 – 5t, vx = = 12t – 5, ax = = 12 0
dt dt
I = 159 N.s
dy dv y
y= 4t2 +3, vy = = 8t, ay = =8
dt dt
dz dv 10 Para la cuerda atada en ambos extremos. Encuentre a) la
z = 3t4 – 3t +1, vz = = 12t3 – 3, az = z = 36t2 tensión en los puntos extremos (b) la tensión en el punto
dt dt
medio.
para t = 1/3 s. tenemos:
ax = 12 m/s2 , ay = 8 m/s2 , az = 4 m/s2  

a = 12 i + 8 j + 4 k

a= 12 2  8 2  4 2 Figura 3.25 La catenaria


a = 4 14 m/s2 Solución Considerando la mitad derecha de la cuerda,
tenemos el diagrama del cuerpo libre que muestra la figura 3.26
Luego ; F = ma = (2 kg)(4 14 m/s2 )

F = 8 14 N. Ty
T

9. Un bloque de 16 kg de masa está bajo la acción de una Tx
fuerza de valor F1 = 50 + 25t2, donde F1 está dado en Newton To
y "t" en segundos.
½W

37° Figura 3.26 DCL de la mitad de cuerda

Las ecuaciones del equilibrio son:

Figura 3.23 Incrementando la fuerza normal Tcos = To (1)


Tsen = W/2 (2)
To
Si el coeficiente de fricción entre el bloque y la superficie es Dividiendo (1) entre (2) hallamos: cotan =
0,2 Determinar el impulso total sobre el bloque en el W/2
intervalo 0 < t < 3 s. Usar g = 10 m/s2 De donde To = ½ W.cotan y también
N T = ½ W.cosec

f 11 Una bola se sostiene en la posición A de la figura 3.27 por


37°
medio de dos hilos ligeros. Se corta el hilo horizontal y la
mg bola comienza a oscilar como péndulo. B es el punto más a la
F1
derecha que la bola alcanza al oscilar. ¿Cuál es la razón de la
Figura 3.24 DCL de la masa deslizante tensión del hilo de soporte en la posición B a su valor en A
antes de cortarse el hilo horizontal?

Solución El DCL, muestra además de (F1), la fuerza de


fricción f = µN, la normal (N). La fuerza resultante en la
dirección del movimiento es:
 
F = F1cos37° – f (1)
desde que no hay movimiento en la dirección normal a la
superficie se tiene:
A B
N – mg – F1sen37° = 0 (2)
Combinando (1) y (2) se obtiene:
Figura 3.27 Liberando al péndulo
80 Dinámica Daniel Fernández Palma / Raúl Zavala Sánchez

Solución Los diagramas DCL en las posiciones A y B se


muestran en la figura 3.28 To To T

  
T2 T2

 
90° F
T1 mg mg
R
W  antes (1) después(2)
W

DCL(punto A) DCL(punto B)
Figura 3.30 DCL de la masa pendular
Figura 3.28 En equilibrio y en movimiento 13 Una masa parte del origen de coordenadas con una
velocidad vo en la dirección del semieje +X y en ausencia de
fuerzas se mueve hasta chocar en una pantalla en el punto A
En el punto A la masa pendular se encuentra en reposo; las situada en x = 2L. Sin embargo si actúa una fuerza externa F
ecuaciones de equilibrio son constante en la dirección del semieje +Y entre x = L y x = L
+ d, (d < L) hará que la masa se desvíe y choque en la
T2 sen = T1 ; T2 cos  = W pantalla en el punto B. a) Explicar si el tiempo para llegar a
la pantalla es igual o diferente en los casos enunciados: con
De donde la tensión de la cuerda oblicua es:
fuerza o libre de ella. b) Calcular la desviación que
T2 = W/cos experimenta la masa al chocar en la pantalla cuando actúa la
fuerza.
En la posición B las fuerzas W y T2' dan como resultante a la
fuerza de recuperación R que tiende a llevar a la masa Solución: a) Como la fuerza F no tiene componentes en la
pendular hacia la posición de equilibrio. En el DCL para el dirección +X, la velocidad en esta dirección permanece
punto B se puede ver que constante y por tanto el tiempo para alcanzar la pantalla es
igual en ambos casos. b) según el enunciado trazamos el
T2' = W cos siguiente esquema
La razón de las tensiones en B y en A es
y
T2' W cos  B
= = cos2 d
T2 W / cos 
y2
12. Un bloque está suspendido mediante dos cuerdas de igual F 
longitud, como se indica en la figura. 3.29
y1 A
0 x
L L

 
Figura 3.31 La fuerza cambia el movimiento

Si hacemos que para t = 0 el móvil se halla en x = L; en la


Fig 3.29 Cambio en el valor de la tensión región donde actúa esta fuerza, el movimiento será de tipo
parabólico:

Si una de las cuerdas se rompe en el instante t1 calcular las x = L + vot, y = ½ at2


tensiones en las cuerdas inmediatamente antes y después de donde a = F/m. Despejando t en la primera ecuación y
t1. reemplazándola en la segunda se obtiene la ecuación de la
Solución: En los DCL antes y después del corte de la cuerda trayectoria
de la izquierda se muestran las fuerzas correspondientes.
a (x  L) 2
De la figura 3.30(1) se tiene: 2Tocos = mg y (1)
2v o2
mg
To  la dirección de la velocidad al momento de salir de la región
2 cos  de aceleración está dada por la pendiente de la curva en x =
De la Figura 3.30(2) cuando se inicia el movimiento debe L+d:
notarse que:
dy
I : F es resultante de mg y de la tensión T , tg  = = (a/vo2)(x – L) = (F/mvo2)(x– L)
dx
II: F es perpendicular a T porque F es la fuerza tangencial en en x = L + d (punto de salida del proyectil) hallamos la
el movimiento pendular. Por tanto: tangente del ángulo:
T = mgcos
Daniel Fernández Palma / Raúl Zavala Sánchez Dinámica 81

Fd Fa – f = ma  Fa = f + ma
tg  2
(2)
mv o Fa = 1,81+8(0,22) = 3,6N

En la parte derecha de la figura 3.31 se ve que:


16. Un trabajador empuja una caja pequeña de 10 kg por una
y 2  y1
tg  (3) superficie horizontal con una rapidez constante de 5,0 m/s.
Ld El coeficiente de fricción cinética entre la caja y el piso es
0,2 a) ¿qué fuerza horizontal debe aplicar el trabajador para
donde y1 es la desviación al momento de salir del campo de
mantener el movimiento?. b) Si se elimina la fuerza, ¿en
fuerzas luego :
cuánto tiempo se detiene la caja?
y1 = ½ (F/m)(d/vo)2 (4)
Solución. (a) Según el enunciado, la caja se mueve a rapidez
Combinando (2), (3) y (4) y despejando y2: constante, lo que significa que no hay fuerza neta actuando
sobre la caja
y2 = ½(F/m)(2L – d)d/vo2
Fa
v
m
f
Fricción
14.Una silla se desliza sobre un suelo pulido, su velocidad
inicial es de 3 m/s. Se detiene después de recorrer 2 m. ¿Cuál Figura 3.33 La fricción se opone al movimiento
es el coeficiente de fricción cinética entre la silla y el piso?
Solución En el movimiento uniformemente acelerado la Por consiguiente la fuerza aplicada Fa es anulada por la
aceleración está dada por fuerza de fricción. Esto es:

v 2  v o2 Fa = f = N = mg = 0,2 (10)(9,8) = 19,6 N


a=
2x (b) Al suprimir Fa, aun queda actuante la fuerza f que
producirá la aceleración en sentido opuesto al movimiento
donde reemplazamos: x = 2 m; vo = 3 m/s, velocidad final v = 0
–f = ma  a = – f /m = –19,6 /10 = –1,96 m/s2
03 2
Para determinar el tiempo necesario para detenerse usamos
a= = -2,25 m/s2
2(2) la ecuación v = vo + at donde v = 0; vo = 5 m/s, a = –1,96
m/s2. Luego:
Desde que la fuerza de frenamiento es la fuerza de
rozamiento: tenemos –f = ma; pero sabemos que f = N = mg vo 5
t= = = 2,55 s
f  a 1,96
a=– = – g
m
17. Un bloque está sobre un plano inclinado cuyo ángulo
de donde hallamos;
puede variarse. El ángulo se incrementa gradualmente desde
a 2,25 0°; a los 30° el bloque comienza a descender por el plano
=– =– = 0,23 inclinado y recorre 3 m en 2 s. Calcular los coeficientes de
g 9,8 fricción estático y cinético entre el bloque y el plano inclinado.
15. En un experimento una caja de 8 kg es empujada sobre N
una mesa plana por una fuerza horizontal F a) si la caja se f
mueve a velocidad constante de 0,350 m/s y el coeficiente de
fricción cinética es 0,14, ¿qué magnitud tiene F? b) y si la mgsen
caja tiene aceleración constante de 0,22 m/s2. c) ¿cómo
cambiarían sus respuestas a (a) y (b) si el experimento fuera
en la Luna (g = 1,62 m/s2) mgcos

Fa v
m Fig 3.34 Deslizamiento con fricción

Figura 3.32 Resultante nula Solución. Cuando el movimiento es inminente;


µs = tg = tg30°  µs = 0,577
Solución a) Si el desplazamiento del móvil es a velocidad El movimiento subsecuente es un movimiento uniformemente
constante, la fuerza neta es nula, de allí que: acelerado con velocidad inicial nula y por tanto su
aceleración es:
Fa – f = 0 ó Fa = f = mg = 0,14(8)(9,8) N
2x
f = 10,946 N a ; x = 3 m; t=2s
t2
b) Fa – f = ma  Fa = f + ma = 10,946 + 8(0,22) Fa= 12,736 N
c) Fa – f = 0  Fa = f = mgL = 0,14(8)(1,62) f = 1,81 N a = 1,5 m/s2
82 Dinámica Daniel Fernández Palma / Raúl Zavala Sánchez

En estas condiciones las ecuaciones dinámicas son para un valor de  = k < c, la caja se desplazara a velocidad
constante cuando la fuerza Fa de la ecuación (4) cumpla con
mg.sen30° - µkN = ma (1)
la siguiente relación:
N = mgcos30° (2)
Fx – Wt – k(Fy +Wn) = 0
gsen30 o  a
Y de ambas: µk = = 0,4 Facos – mg.sen = k (mg.cos  + Fasen), de donde:
g cos 30 o
mg(sen   k cos )
Fa =
cos    k sen
18. Una mujer trata de empujar una caja para subirla por una
rampa que forma un ángulo  sobre la horizontal. Los
coeficientes de fricción entre la rampa y la caja son s y k. 19 Con una fuerza Fo de magnitud constante e igual a 50N
La fuerza F aplicada por la mujer es horizontal a) si s es pero dirección variable dada por el ángulo  se comunica un
mayor que cierto valor critico c la mujer no puede poner en impulso al bloque en dirección horizontal. Si el coeficiente
movimiento la caja, por mucho que empuje. Calcule dicho de fricción entre el bloque y la superficie es µ = 5/12.
valor crítico, b) suponga que s es menor que el valor crítico. Calcular el valor de  para el impulso máximo en el
¿qué magnitud de fuerza debe aplicar la mujer, para mantener intervalo 0 < t < 3 s.
la caja subiendo con rapidez constante?
Solución
Solución Según el enunciado el diagrama de fuerzas sobre el
bloque se muestra en la figura 3.35 donde Fx y Fy son las En el eje X: Resultante = F
componentes de la fuerza aplicada Fa; Wt y Wn son las
Focos – f = F (1)
componentes de la fuerza peso, N la fuerza de contacto y f la
fuerza de rozamiento: En el eje Y: Resultante = 0
N + Fosen – mg = 0 (2)
N
Fx combinando (1) y (2), y usando f = µN, se tiene el valor de la
Fa fuerza resultante F:

m F = Fo(cos + µsen) – µmg
Fy El impulso máximo se obtendrá para una resultante máxima:
Luego aplicamos dF/d = 0 (condición para máximos y
Wt f Wn mínimos)

d
[Fo(cos + µsen) – µmg] = 0
Figura 3.35 Cómo subir una caja d
– sen + µcos = 0
Resultante en la dirección del plano inclinado: Así hallamos: tg = µ = 5/12. Es decir existe un máximo
cuando  = arctg(5/12)
Fx – Wt – f = ma (1)
Resultante en la dirección normal al plano inclinado El impulso es: I =  Fdt = Ft
N – F y – Wn = 0 (2)
I = [Fo(cos +sen)– mg]t
En esta última ecuación hallamos N = F y + Wn y utilizando f
= N obtenemos: Si tan  = 5/12, podemos encontrar: cos  = 12/13; sen  =
5/13, y utilizando Fo = 50 N , m = 12 kg, t = 3 s hallamos:
f = (Fy +Wn) (3)
I = 12,5 N.s
Reemplazando en (1) se tiene:
Fx – Wt – (Fy +Wn) = ma (4)
20. Un objeto de 2 kg de masa se lanza hacia arriba con una
En esta ecuación se puede ver que si admitimos que  es velocidad de 40 m/s sobre un plano inclinado 20°. Si el
variable, el movimiento ascendente será inminente (o coeficiente de rozamiento es 0,2 determinar la máxima
posible) para un valor de  = c tal que: distancia que subirá sobre el plano y la velocidad del objeto
cuando regresa al punto de partida.
Fx – Wt – c (Fy +Wn) = 0
Solución Las componentes del peso en las direcciones
Luego para cualquier valor de  = s > c será imposible el paralela y perpendicular al plano inclinado son:
movimiento ascendente ya que según la ecuación (4) la
fuerza neta sería negativa. El valor crítico del coeficiente de w' = mg.sen  (1)
rozamiento es: w" = mg.cos  (2)

Fa cos   mgsen  fuerza de fricción.


c =
Fa sen  mg cos  f = µN (3)
Para escribir las ecuaciones dinámicas elegimos el eje +X
paralelo al plano inclinado y hacia arriba
Daniel Fernández Palma / Raúl Zavala Sánchez Dinámica 83

A) Cuando el cuerpo asciende:


F
Fix : – f – w' = ma1 (4)
Fiy : N – w" = 0 ó N = w" (5) W
De modo que f = µN = µw" = µmg.cos. Luego la ecuación
(4) se transforma a: mgsen

– µmg.cos – mg.sen = ma1
de donde la aceleración es: Figura 3.36 Deslizamiento sobre un plano inclinado
a1 = – g(sen + µcos)
Con esta aceleración calculamos la distancia recorrida hasta En el plano inclinado: (no hay rozamiento)
que la velocidad se reduce a cero:
F = Wsen30° (1)
v 2  v 02 0  v 02
d  En el plano horizontal:
2a1  2g(sen   cos )
F=f'+f (2)

v 02
d (6) W' W F
2g(sen   cos )

f f
B) Cuando el cuerpo desciende;
– w' + f = ma2 Figura 3.37 Las fricciones se suman

reemplazando valores resulta:


donde f ' = µW' y f = µW; luego igualando (2) y (1) y
– mg.sen + µmg.cos = ma2
despejando W' tenemos:
lo que nos da:
Wsen30 o  W
a2 = – g(sen – µcos) (7) W' 

En el movimiento de descenso la velocidad inicial es cero de
modo que después de recorrer la distancia d, su velocidad es: W' = W

v = √2|a2 |d
Problemas con dos o más cuerpos
v  2dg(sen   cos) (8)

utilizando el valor de d dado por (6) tenemos: 22. En la figura se muestra las masas m1 y m2 cuyos pesos
son respectivamente 5 N y 45 N. Si el sistema está
sen   cos
v  vo inicialmente en reposo y no se toma en cuenta la fricción en
sen   cos la polea, calcular la velocidad con que m2 llega al piso
cuando se suelta a m1; h = 5m
con vo = 40 m/s,  = 20° y µ = 0,2 tenemos:
m sen20o  0,2 cos 20o
v  40
s sen20o  0,2 cos 20o

v = 21,57 m/s
m2

21. Un motor impulsa a velocidad constante vo a un peso W


hacia arriba de un plano inclinado 30° con la horizontal. No h = 5m
hay fricción entre el peso y el plano inclinado. m1
Horizontalmente, el mismo motor y a la velocidad vo, puede
arrastrar además del peso W, otro W'. Si en este caso el
coeficiente de fricción es 0,25. Determinar el valor de W'.
Figura 3.38 Máquina de Atwood
Solución. Como la velocidad en ambos casos es la misma; la
potencia (P = Fv) del motor no cambia por consiguiente la Solución: Con los diagramas (DCL) de cada masa,
fuerza (F) es constante. calculamos la aceleración del sistema:
84 Dinámica Daniel Fernández Palma / Raúl Zavala Sánchez

tensión en Q (TQ) por 6 N. Calcular: a) La aceleración del


T T ascensor. b) Las tensiones en los puntos P, Q y R.(usar g =
+a 10 m/s2)
m1 m2 -a

m1g m2g
P
Q
Figura 3.39 DCL de las masas m1 y m2
A 10 kg
a
Las ecuaciones según los DCL mostrados por la figura 3.39 R
son:
T – m1g = m1a (1) B 20 kg
T – m2g = – m2a (2)
de (1) y (2) la aceleración buscada es:
(m 2  m1 ) Figura 3.40 cuerdas con masa no despreciable
a g (3)
m 2  m1

Desde que el peso de un cuerpo es proporcional a su masa la Solución Sean: mC = masa de cada cuerda; mA, mB = masas
ecuación (3) se transforma a: de los bloques
Fuerza resultante sobre el punto P:
(w 2  w1 )g
a (4)
w 2  w1 TP – (2mC+mA+mB)g = (2mC+mA+mB)a (1)
Fuerza resultante sobre el punto Q:
Reemplazando valores hallamos: a = 7,84 m/s2. De: v2 = vo2
+ 2ah, con vo = 0 tenemos: TQ - (mA+ mC+ mB)g = (mA+mC+ mB)a (2)

v= 2ah = 2(7,84)(5) = 8,85 m/s. Fuerza resultante sobre el punto R:


TR – (mC + mB)g = (mC + mB)a (3)
23. Una masa m1 de 100 kg es empujada a lo largo de una
superficie sin rozamiento por una fuerza F de tal modo que Al restar (2) de (1) miembro a miembro se obtiene TP – TQ –
su aceleración es 5 m/s2. Una masa m2 de 20 kg se desliza mCg = mCa. Por tanto:
por la parte superior de la masa m1 con una aceleración de 3
m/s2 ( por tanto se desliza hacia atrás respecto a la masa m1). TP  TQ  m C g
a
a) Cuál es la fuerza de fricción ejercida por la masa m1 sobre mC
la masa m2?. b) ¿Cuál es la fuerza neta que actúa sobre la
masa m1?. c)¿Cuánto vale la fuerza F? d) Una vez que la donde TP – TQ = 6 N, mC = 0,5 kg, g = 10 m/s2
masa m2 se ha caído de la masa m1 ¿cuál es la aceleración
Luego el resultado numérico para el valor de la aceleración
que adquiere esta última?
es: a = 2 m/s2
b) De (2): TQ = (mA + mC + mB)(g + a)
m2 3 m/s2 TQ = (10 + 0,5 + 20)(10 + 2)
5 m/s2 TQ = 366 N
m1 TP = TQ + 6 = 372 N
F
De (3) TR = (mC + mB)(g + a)
TR = (0,5 + 20)(10 + 2) = 246 N
Figura 3.41 Deslizamiento TR = 246 N
Solución a) fuerza de fricción entre m1 y m2 está dado por f =
m2 a2 = (20)(3) = 60 N
25. Con los datos de la figura, determine la aceleración de los
b) fuerza neta que actúa sobre la masa m1 es: F1 = m1a1 = bloques, si el coeficiente de fricción para todas las superficies
100(5) = 500 N es 0,2 y es nulo en la polea; m1 = 5 kg, m2 = 20 kg
c) valor de la fuerza F es F = (m1 + m2)a1 = (100 +20)(5) =
600 N
d) aceleración de m1 al caer m2: m1
F 600 m2
a1 = = = 6 m/s2
m1 100
30°

Figura 3.42 Dos fuerzas de contacto


24 Dos bloques están sujetos a la parte superior de un
ascensor como se muestra en la figura. Cada cuerda tiene una
masa de 0,5 kg. Si la tensión en P (TP) es mayor que la
Daniel Fernández Palma / Raúl Zavala Sánchez Dinámica 85

Solución Del DCL de la masa m1 se deducen las siguientes Obsérvese que en total tenemos 3 incógnitas T, a1 y a2 y tan
ecuaciones solo 2 ecuaciones. De modo que para la solución del
f1 problema se requiere una ecuación más. Notemos que si m2
N1 N2 T avanza una distancia d la masa m1 tan solo avanza la mitad
T
f2 (d/2). Por tanto las aceleraciones están en la misma
m1gsen proporción:
m1gcos m2g sen (m2 + m1)cos
f1 a1 = ½ a2 (3)
Resolviendo simultáneamente las tres ecuaciones hallamos:

Figura 3.43 DCL ( m1 ) Figura 3.44 DCL ( m2 ) 2m 2 g 4m 2 g m1m2g


a1 = , a2 = ,T=
4m2  m1 4m2  m1 4m2  m1
T – m1gsen – f1 = m1a (1)
N1 – m1gcos = 0 (2)
27. En la figura, un bloque de masa m1 = 10 kg descansa
De la figura 3.44 deducimos las siguientes ecuaciones: sobre un soporte de masa m2 = 5 kg. Los coeficientes de
m2gsen - f1 - f2 - T = m2a (3) fricción entre el bloque y el soporte son s = 0,4 y k = 0,3.
El soporte se apoya sobre una superficie sin rozamiento. a)
N2 – (m1+m2)gcos = 0 (4)
¿Cuál es la fuerza máxima F que puede aplicarse, sin que el
De la definición de fuerzas de fricción tenemos: bloque de 10 kg deslice sobre el soporte?. b) ¿Cuál es la
aceleración correspondiente del soporte?
f1 = N1; f2 =  N2 (5)
F
Con N1 y N2 dados por las ecuaciones (2) y (4); l.as fuerzas
de rozamiento son: m1
f1 = m1g.cos; f2 = (m1 + m2)cos  m2
Con estos resultados y las ecuaciones (1) y (3) se obtiene la
expresión de la aceleración:
Figura 3.47 Masas deslizantes
(m2  m2 )gsen  (3m1  m2 )g cos
a=
m1  m2
Solución Si m1 no desliza sobre el soporte, la fuerza F actúa
reemplazando valores, hallamos: a = 0,56 m/s2
sobre el conjunto bloque y soporte como un sólo objeto. por
26. Un bloque de masa m1 = 20 kg dotado de una polea, se consiguiente escribimos:
desliza a lo largo de una mesa sin rozamiento. Como se
F = (m1 + m2)a (1)
indica en la figura, está conectado mediante una cuerda de
masa despreciable a otro bloque de masa m2 = 5 kg. a) El valor de F está limitado por la fuerza de fricción cuyo
Determinar la aceleración de cada bloque y la tensión en la valor máximo es f = F = sm1g
cuerda que conecta ambos bloques
F = 0,4(10)(9,8) = 39,2 N (2)
m1 b) Reemplazando (2) en (1) se tiene:
F 39,2
a= = = 2,61 m/s2
m2 m1  m 2 10  5

28. Dos masas de 50 kg cada una son arrastradas a lo largo de


una superficie pulida con una aceleración constante de 1 m/s2
Figura 3.45 Aceleración duplicada Cada una de las cuerdas utilizadas tiene una masa de 1 kg tal
como se indica en la figura. Calcular la fuerza F y las
Solución El DCL para cada uno de los bloques sin tomar en tensiones en las cuerdas en los puntos A, B y C
cuenta el peso de m1 y su reacción respectiva se muestra en la
siguiente figura C B A
F
T 50 kg 50 kg
T
m1 m2
T
m2g Figura 3.48 Cuerdas y masas aceleradas

Figura 3.46 DCL de m1 y m2


Solución Aplicando la 2da ley de Newton al conjunto.
F= ma = (50kg + 1kg + 50kg +1kg)(1m/s2)
Si m2 desciende las ecuaciones respectivas son: F = 102 N.
2T = m1a1 (1) En el punto A, la, tensión TA es la fuerza resultante que
T– m2g = – m2a2 (2) acelera a todas las masas que quedan a la izquierda de A; por
tanto:
86 Dinámica Daniel Fernández Palma / Raúl Zavala Sánchez

TA = (50kg +1kg +50kg)(1m/s2) = 101 N. Solución. Designando con T la tensión entre m1 y m2 y con T '
la tensión entre m2 y m3. Los DCL se muestran en las tres
análogamente en los puntos B y C. figuras siguientes:
TB = (50 kg + 1kg)(1m/s2) = 51 N
TC = (50 kg)(1 m/s2) = 50 N. T T' T'

m1 m2 m3
29. Una fuerza P = 150 N se aplica al conjunto de bloques A
y B de manera que se mueva sobre la superficie del plano m2g
como se indica en la figura 3.49, si se considera que no hay T
fricción; calcular la fuerza entre los bloques si sus masas
m1g m3g
son mA = 4 kg y mB = 2 kg. (a) (b) (c)

Figura 3.52 DCL para cada uno


B
A

P Considerando el movimiento descendente de m3, las


ecuaciones dinámicas son:
30°
T – m1g = m1a (1)
Figura 3.49 Fuerza de contacto en lugar de tensión T ' – T – m2g = m2a (2)
T ' – m3g = – m3a (3)
Solución. En la dirección del plano inclinado tenemos las
siguientes fuerzas: la fuerza P hacia arriba del plano Cambiando de signo a (3) y sumando miembro a miembro
inclinado y las componentes de los pesos en dirección (1), (2) y (3)
opuesta: (m3 – m1 – m2)g = (m1 + m2 + m3)a
Aplicamos entonces:  Fj = ma; donde m = mA + mB despejando el valor de a se tiene:
P – mAgsen30° – mBgsen30° = (mA + mB)a m 3  m1  m 2
150 – 4(9,8)(1/2) – 2(9,8)(1/2) = (4 + 2)a, a g
m 3  m 2  m1
a = 20 m/s2
De (1) la tensión T es: T = m1g + m1a
Escribiendo la ecuación dinámica sólo para la masa A y
considerando la fuerza FBA (fuerza de contacto) que es la 2m1m 3g
fuerza que ejerce el bloque B sobre el bloque A T
m1  m 2  m 3
FBA y de (3) se tiene:

A 2m 3 (m1  m 2 )g
T' 
m1  m 2  m 3

P mAg sen  numéricamente:


1 28 84
Figura 3.50 FBA fuerza de contacto a= g, T= g, T' = g
13 13 13
31. Dos objetos de masas m1 y m2 deslizan hacia abajo sobre
P – mAgsen30° – FBA = mAa un plano inclinado sin fricción y con un ángulo  con la
horizontal, como se ve en la figura.
Despejando la fuerza requerida:
FBA = 50,4 N m1

m2
30. En la Fig 3.51 hallar: a) La aceleración de las tres masas
b) La tensión en cada cuerda. m1 = 2 kg, m2 = 4 kg, m3 = 7 kg.

Figura 3.53 Deslizamiento conjunto


En la superficie de contacto entre los dos cuerpos hay una
m2 fuerza de fricción f suficiente para impedir el deslizamiento
de uno sobre el otro. Hallar: a) La aceleración del sistema. b)
La fuerza normal sobre el plano Inclinado c) La fuerza
m3 normal o fuerza de contacto entre las dos masas d) La fuerza
m1 de fricción en la superficie de contacto entre los dos cuerpos.
Solución. a) La componente del peso total en la dirección
(descendente) del plano inclinado acelera al par de cuerpos
Figura 3.51 Tensiones diferentes para cada cuerda
Daniel Fernández Palma / Raúl Zavala Sánchez Dinámica 87

Si no existe rozamiento entre el piso y el bloque B. Calcular:


m1 a) las aceleraciones de A y B b) la reacción que ejerce el piso
N2 sobre el bloque B.
Solución: Las componentes horizontales de la acción y la
m2 reacción entre A y B ocasionan el movimiento de B hacia la
derecha y la de A hacia la izquierda. Las ecuaciones
dinámicas, se obtienen del DCL de cada bloque

W' W'' N'
N
(m1+ m2)g

Figura 3.54 DCL



 
Ecuaciones: N2 – W'' = 0 (1) N
W' = (m1 + m2)a (2)
donde W' y W'' son componentes del peso total: mAg mBg

W'' = (m1 + m2)gcos


Figura 3.57 DCL para cada móvil
W' = (m1 + m2) g sen
y por tanto de las ecuaciones (1) y (2) obtenemos
respectivamente: Para el bloque A:
N2 = (m1 + m2)gcos – N.cos(90°– ) = mA.ẍ A (1)
a = g.sen – mAg + N sen(90°– ) = mA. ÿ A (2)
c) La masa m1 al igual que m2 tiene aceleración a = g.sen Para el bloque B :
paralela al plano inclinado; la componente de ésta
aceleración en la dirección vertical es: Ncos(90°- ) = mB ẍ B (3)
N' – Nsen(90°– ) – mBg = 0 (4)
ay = a.sen = g.sen2
La ecuación dinámica para m1 es:
Combinando (1) y (2) obtenemos:
N – m1g = – m1ay
ÿ A = – g – ẍ A ctg (5)
Así, el valor de N es: N = m1g(1 – sen2) = m1g.cos2
En un sistema de referencia fijo en B, las componentes de la
d) fuerza de fricción sobre la masa m1 aceleración de A respecto a B, son tales que:
ÿ A⁄B ÿ A −ÿ B
N = = tan 
ẍ A⁄B ẍ A −ẍ B

m1 Donde ÿ B = 0, puesto que B no tiene movimiento vertical:


f ÿ A = ( ẍ A - ẍ B ) tan  (6)
Nótese que los segundos miembros de las ecuaciones (5) y
m1g (6) deben ser iguales. Esto es
( ẍ A - ẍ B ) tan  = - g - ẍ A ctan (7)
Figura 3.55 DCL de la masa m1 De (1) y (3) obtenemos:
mA ẍ A
ẍ B = – (8)
mB
f = µN = µm1 gcos2
Con estos resultados hallamos
− g sen θ.cosθ
32 Un bloque de masa mA se desliza sin fricción sobre otro ẍ A =
1 + (mA ⁄mB )sen2 θ
bloque de masa mB como se indica en la figura.
− (mA +mB )g sen2 θ
ÿ A =
mB + mA sen2 θ
g sen  cos 
mA ẍ B =
sen2 θ+ (mB ⁄mA )

mB b) Por tanto la reacción del piso mediante (2) y (4) es:



(m A  m B )m B g
N' 
Figura 3.56 Plano inclinado móvil m B  m A sen 2 
88 Dinámica Daniel Fernández Palma / Raúl Zavala Sánchez

33. En el dispositivo mostrado en la figura 3.58, la masa m se Si N = 0 la fuerza de fricción es nula de acuerdo con la
mantiene en reposo respecto a la masa M, mediante la fuerza ecuación (4). Luego, de (6) se tiene que:
de fricción estática máxima (coeficiente µs) y además por un
mg
tope que la retiene. Determinar la fuerza ejercida por dicho FT  (11)
tope. ¿Cuál será el valor de la fuerza F necesaria para que la sen
normal a la superficie de contacto entre las masas sea nula.
En este caso ¿Qué valor tiene la fuerza de fricción? ¿Qué 34. Determinar la aceleración de los bloques A y B de masas
valor tiene la fuerza del tope? mA = 100kg y mB = 40 kg y la tensión en la cuerda que
sujeta a B (rozamientos y masas de las poleas nulos) Usar g
= 10 m/s2

M

Figura 3.58 Anulando la fuerza de contacto


mB
mA
Solución: La fuerza resultante sobre el sistema formado por
las masas M y m es F por tanto:
Figura 3.60 Una pesa más rápida que la otra
F
a (1) Solución Obsérvese que si se desea que la polea móvil
mM descienda una distancia h, es necesario incrementar la cuerda
Para determinar la fuerza FT que ejerce el tope sobre la masa que lo sostiene en 2h. De allí que el desplazamiento del
m, usamos el diagrama correspondiente y escribimos: bloque A es la mitad del desplazamiento de B. Las
velocidades y aceleraciones también guardan la misma
(FT + f )cos – Ncos(90°– ) = ma (2) relación. aA = a y aB = 2a
(FT + f )sen – Nsen(90°–) – mg = 0 (3)
donde f es la fuerza de fricción T T
T
f = µs N (4)

N f mB
90°- 
mA
mBg
mAg

FT mg
Figura 3.61 DCL de las masas mA y mB
Figura 3.59 DCL Las ecuaciones dinámicas del movimiento son:
2T – mAg = – mA(a) (1)
Introduciendo en (2) y (3) el valor de f T – mBg = mB(2a) (2)
FTcos – N(µscos – sen) = ma (5) Resolviendo simultáneamente (1) y (2) hallamos las
FTsen + N(µssen + cos) = mg (6) aceleraciones aA = a y aB = 2a y la tensión T en la cuerda.
que pueden resolverse simultáneamente para FT y N ma  2mB 3mA mBg
obteniéndose: a g; T 
mA  4mB m A  4m B
FT = ma(µssen + cos) + mg(sen – µscos) (7)
N = mg.cos – ma.sen (8) numéricamente:
g
a ; T = 46,1 N
Si a = 0, FT = mg(sen – µscos), 13
N = mgcos

Fuerzas ficticias y movimiento circular


Si N = 0, en la ecuación (8), la aceleración tiene el siguiente valor:
35. Un hombre se encuentra sobre una balanza situada en un
a = g.ctg (9)
ascensor que posee una aceleración ascendente a. La escala
y reemplazando en (1) hallamos la expresión para F: de la balanza marca 960 N . Al coger una caja de 20 kg, la
escala marca 1200 N. Calcular la masa del hombre, su peso y
F = (M + m)g.ctg (10) la aceleración a
Daniel Fernández Palma / Raúl Zavala Sánchez Dinámica 89

Solución El hombre dentro del ascensor está bajo la acción Como fs = sN = s(mg), hallamos:
de las siguientes fuerzas: su peso W = mg; la reacción del
piso del ascensor F cuyo valor se lee en la balanza: fs 0,643
s = = = 0,64
mg (0,1)(9,8)
F - mg = ma (1)
Según el enunciado en la primera vez F = 960. Entonces 37. Una carretera está peraltada de tal modo que un coche
desplazándose a 40 km/h puede tomar una curva de 30 m de
960 - mg = ma (2) radio incluso si existe una capa hielo equivalente a un
coeficiente de fricción aproximadamente igual a cero.
Determinar el intervalo de velocidades a que un coche puede
tomar esta curva sin patinar, si el coeficiente de fricción
estática entre la carretera y las ruedas es de 0,3
Solución

N

Fc
Figura 3.62 Peso dentro del ascensor
Al incrementarse la masa en 20 kg, la fuerza F aumenta a mg
1200 N. Por consiguiente la ecuación (1) nos da para este
caso: 
1200 – (m+20)g = (m+20)a (3)
Figura 3.64 Fuerza centrífuga en una curva peraltada
resolviendo simultáneamente (2) y (3) se obtiene:
m = 80 kg; a = 12 – g = 2,2 m/s2
Determinamos previamente el ángulo de peralte asumiendo
peso W = mg = 784 N que no hay fricción. En la figura 3.64 el equilibrio de fuerzas
nos da:

36. Un disco de 100 g se coloca sobre una plataforma N.sen = Fc


giratoria horizontal que gira a razón de 1 rev/s. El disco está Ncos  = mg
situado a 10 cm del eje de rotación de la platafoma. a) ¿Qué Dividiendo miembro a miembro se encuentra tan = Fc/mg,
fuerza de fricción actúa sobre el disco? B) El disco desliza y donde Fc = mv2/r
sale despedido de la plataforma cuando se coloca a una
distancia radial superior a 16 cm del eje de rotación. ¿Cuál es v2 (11,1) 2
el coeficiente de fricción estática? tan = = = 0,42
gr (9,8)(30)
Solución El disco se mantiene a distancia fija del eje de la
plataforma a causa de la fuerza de fricción que anula a la  = 22,8°
fuerza centrifuga fc
Consideremos ahora la fricción y admitiendo que el coche
fc = m(42f2)r tiende a patinar hacia abajo, la fuerza de rozamiento apunta
hacia arriba, como se muestra en el DCL siguiente
siendo m = 100g = 0,1 kg y f la frecuencia de rotación f = 1
rev/s.
A la distancia r = 10 cm = 0,10 m la fuerza de fricción es fs =
N
fs
(0,1)(4)(3,14)2(1)2(0,1) = 0,394 N 
 Fc

 mg
v

r Figura 3.65 DCL


plataforma rotante
Las ecuaciones en una posición instantánea son
Fc + fs cos = N sen  (1)
Figura 3.63 Fuerza ficticia en la plataforma rotante fs sen + N cos  = mg (2)
si reemplazamos fs = sN en (2) hallamos:
El valor límite de la fuerza de fricción ocurre cuando r = 16 mg
cm = 0,16 N= (3)
cos    s sen
fs = (0,1)(4)(3,14)2(1)2(0,16) = 0,643 N
90 Dinámica Daniel Fernández Palma / Raúl Zavala Sánchez

Obsérvese que Fc = mv2/R (fuerza centrífuga) Luego Solución a) Si el giro tiene lugar en el sentido indicado por la
reemplazando (3) en (1) encontramos la velocidad mínima v flecha (sentido de rotación de las agujas del reloj) la amiga
= v1 experimentará la acción de la fuerza centrifuga y será
impulsada hacia el conductor.
 sen   s cos  
v1 = gR 
 (4)
 cos    s sen 

o también

 tan   s 
v1 = gR  
 (5)
 1  s tan   Figura 3.67
Para encontrar el límite superior admitimos que el coche
tiende a patinar hacia arriba. Por consiguiente la fuerza de b) Si la velocidad es constante la fuerza centrípeta (o
fricción apunta hacia abajo y en las ecuaciones debemos centrifuga) depende inversamente del radio de giro. Por
escribir - fs en lugar de fs o directamente en (5) reemplazar s consiguiente la atracción de la amiga hacia el conductor será
por -s. El resultado es: efectiva hasta un radio para el cual la fuerza centrifuga sea
equilibrada por la fuerza de fricción. En tal caso se tiene
 tan    s  fuerza centrifuga = fuerza fricción
v2 = gR  

 1   s tan   v2
m = smg
Los resultados numéricos se obtienen con g = 9,8 m/s 2, R= r
30 m , tan = 0,42, s = 0,3
de donde: r = v2/sg; Reemplazando v = 25 m/s y s = 0,4 se
v1 = 5,6 m/s, v2 = 15,6 m/s tiene:
r = 159 m
38 ¿Qué aceleración debe tener el carrito de la figura 3.66
para que el bloque A no se caiga?. El coeficiente de fricción 40 Un peso de masa m está sujeto mediante dos cuerdas AC
estática entre el bloque y el carrito es s ¿Cómo describiría y BC como se muestra en la figura 3.68.
un observador en el carrito el comportamiento del bloque?
A

mA h
mB B
 m
a R C

Figura 3.66 mA bajo la acción de una fuerza ficticia


Solución Al acelerar el carrito con una aceleración a, se
produce entre A y B una fuerza de contacto N que da lugar a
la fricción entre las superficies que mantendrá a la masa A Figura 3.68 Un péndulo cónico
sin caerse. Esto es la fuerza que equilibra al peso de A es: La masa m se hace girar en una circunferencia horizontal
f = mg = sN (1) con una velocidad angular  y  = 60°,  = 30°. Determinar
las tensiones en las cuerdas TBC = T1, y TAC = T2.
Como N es la única fuerza que obra sobre mA le comunica la
aceleración a Solución: En el DCL de la masa rotante se muestra:

N = ma (2) - la tensión en la cuerda BC…….= T1


- la tensión en la cuerda AC…….= T2
De (1) y (2) se tiene: - el peso de la masa m ………….= mg
g T2
a=
s T1

39 Un joven conduce con su amiga sentada en el asiento de mg


copiloto, y decide usar la física para tenerla más cerca dando
una vuelta rápida. a) ¿debe girar el auto a la derecha o a la Figura 3.69 DCL de la masa rotante
izquierda para lograr su romántico objetivo? b) Si el
coeficiente de fricción estática entre la amiga y el asiento es
de 0,40 y el auto viaja a 25 m/s (constante). ¿con qué radio
La resultante de estas tres fuerzas es la fuerza centrípeta: Fc
máximo de la curva la amiga todavía se desliza hacia el
joven? T1 + T 2 + m g = Fc
Daniel Fernández Palma / Raúl Zavala Sánchez Dinámica 91

Dado que la fuerza centrípeta es horizontal la expresión


anterior escrita en componentes escalares conduce a:
N
T1 cos30° + T2 cos60° = Fc, 
T1 sen30° + T2 sen60° = mg
Recordando que la fuerza centrípeta está dada por Fc = m2R Fc
y observando en la figura 3.68 que el radio R y la altura h
están relacionados por
mg
tg  = tg30° = R/h; ó R = h/√3
las ecuaciones simultáneas a resolver son: Figura3.71 DCL de la cuenta
2
√3T1 + T2 = m.2.h
√3
T1 + √3T2 = 2.mg b) Si la cuenta se mantuviera a la altura del centro del aro, el
ángulo  tendría el valor de 90° y cos = 0, por tanto la
de donde las soluciones son: ecuación (2) sería absurda ya que sólo el primer miembro
3g−ω2 h sería cero y no así el segundo. Esto significa que en ausencia
T1 = m2h – mg ; T2 = m de rozamiento entre el aro y la cuenta, no es posible la
√3
ubicación de la cuenta a la altura del centro del aro
c) si la frecuencia es f = 1 rev/s se tiene:
41 Una cuenta de un collar puede deslizarse sin fricción por
un aro circular en un plano vertical con radio de 0,100 m. El cos = 2,48 ??
aro gira a razón constante de 3,00 rev/s sobre un diámetro Se sabe que el coseno de un ángulo es menor o igual que 1.
vertical. a) Calcule el ángulo  en que la cuenta está en Por tanto en la ecuación (4) tenemos:
equilibrio vertical (desde luego tiene la aceleración radial
hacia el eje. b) ¿puede la cuenta mantenerse a la misma altura g
cos = 1
que el centro del aro? c) ¿qué sucede si el aro gira a 1.00 4 2 f 2 R
rev/s ?
Esto es, el valor de la frecuencia debe ser superior o igual a
 cierto valor crítico. Esto es:

1 g
f  = 1,58 rev/s
2 R

R 
Fuerzas de resistencia
42. Newton demostró que la resistencia que ofrece el aire a
r
un objeto de sección transversal circular de radio r en caída
libre es aproximadamente igual a ½r2v2 donde  = 1,2
kg/m3 es la densidad del aire. Calcular la velocidad límite de
Figura 3.70 Aro rotante un paracaidista de 56 kg si se supone que su sección
transversal es equivalente a la de un disco de 0,30 m de radio

Solución En una posición instantánea las fuerzas que actúan Solución La velocidad límite resulta en el instante en que la
sobre la cuenta son: N, fuerza radial de reacción o normal fuerza de resistencia del aire es equilibrada por el peso del
que mantiene en contacto al aro y la cuenta, W = mg el peso cuerpo Fd = mg:
de la cuenta, Fc la fuerza centrífuga ½r2v2 = mg
Las ecuaciones dinámicas son de donde despejamos y aplicamos :
N sen  = Fc (1)
2mg 2(56)(9,8)
N cós  = mg (2) v = vL = = = 57 m/s
r 2 1,2(3,14)(0,30)2
Donde Fc = mv2/r = m(42f2)r; r = R sem 
Fc = m(42f2)R sen  (3)
43. Un automóvil de 800 kg desciende por una larga
Dividiendo miembro a miembro (1) y (2) y utilizando (3) se
pendiente de 8°. La fuerza de arrastre que se opone al
obtiene:
movimiento del coche tiene en SI la forma Fd = 100 + 1,2v2.
g ¿Cuál es la velocidad terminal del automóvil al descender por
cos = (4) esta pendiente?
4 2 f 2 R
Solución La fuerza que provoca el movimiento en la
sustituyendo f = 3 rev/s = 3 s-1, R = 0,1 m, g = 9,8 m/s2 se dirección descendente sobre el plano inclinado es la
tiene: componente del peso: Wx = mg.sen. A esta fuerza se opone
cos = 0,2758;  = 74° la fuerza de arrastre Fd; de modo que al equilibrarse ambas
resulta la velocidad límite. Entonces
92 Dinámica Daniel Fernández Palma / Raúl Zavala Sánchez

100 + 1,2v2. = mg.sen ¿Cuáles son la dirección y magnitud de la aceleración del


objeto?. Si hubiéramos especificado las fuerzas en un orden
de donde: diferente, habría sido diferente la respuesta?
mgsen  100 800(9,8)sen8  100 4. Un automóvil tiene 1300 kg de masa y acelera desde el
v= = reposo hasta 81 km/h en 16 segundos. Agregando un perfil
1,2 1,2
aerodinámico, el mismo vehículo puede acelerar a la misma
v = 28,7 m/s velocidad en 12 segundos. ¿Cuál es la diferencia entre las
fuerzas que ejerce el aire (la resistencia del aire), en el
automóvil en los dos casos?
44 Se libera una pelota desde una gran altura. En términos de 5. Una fuerza común de contacto es la que da una cuerda
g diga, que componente "y" de la aceleración tiene cuando su tensa. Suponga que una cuerda tensa esta fija a un cuerpo.
rapidez es la mitad de la velocidad terminal sabiendo que el ¿En qué dirección puede esa cuerda aplicar una fuerza al
arrastre es proporcional a v2 cuerpo? A esta fuerza se llama tensión (a) Se jala un carrito
de 8,5 kg de masa en dirección horizontal con una cuerda
ligera, de masa despreciable y acelera en dirección horizontal
Fd a 2,6 m/s2 ¿Cuál es la tensión de la cuerda?. Si el mismo
carrito se jala ahora con una aceleración de 0,85 m/s2, ¿Cuál
v es la tensión?. Suponga que la única fuerza que actúa sobre el
carrito es la tensión. b) Suponga que la cuerda pasa por una
polea fija y que el carrito acelera horizontalmente a 3,5 m/s2.
mg ¿Cuál es la fuerza vertical hacia arriba que se ejerce sobre la
polea?
Figura 3.72 Pelota en descenso
Tensión

Según el DCL (figura 3.72) tenemos la siguiente ecuación a


para el movimiento:
mg - kv2 = ma (1)
El móvil alcanza la velocidad terminal o límite v L, cuando la
aceleración se reduce a cero es decir la fuerza neta es nula. Figura 3.73 la tensión es el esfuerzo
Luego en la ecuación (1) se encuentra que:

mg
vL = (2) 6. Los electrones cuya masa es 9,110-31 kg son
k constituyentes de los átomos y responden a fuerzas eléctricas.
Estas fuerzas de intensidades variables se pueden generar en
pero cuando la velocidad es aún v = ½ vL; la ecuación (1)
el laboratorio. Suponga que sobre un electrón actúa una
queda expresada así:
fuerza eléctrica constante de 10-15 N. ¿Cuál es la velocidad
mg – k(½vL)2 = ma (3) del electrón después de transcurrido 10-10 s ? ¿Después de 10-9 s ?
¿ Después de 10-7 s ?
de donde despejamos la aceleración:
7. Un gancho está atornillado al techo. Se fija a él una
a = g – ¼k(vL)2/m cuerda cuyo extremo opuesto tiene fija una masa de 10 kg.
y utilizando la velocidad límite ec.(2) hallamos: Suponiendo que cuando actúa la fuerza de gravedad sobre
una masa m, su magnitud es mg, donde g = 9,8 m/s2. ¿Qué
a=¾g fuerza ejerce el gancho sobre la cuerda?
8. En el problema anterior se especificaron las fuerzas que
3.14 PROBLEMAS PROPUESTOS actúan sobre determinados cuerpos. De acuerdo con la
tercera ley de Newton, esos objetos son los orígenes de
1. Las fuerzas que actúan sobre un aeroplano son: gravedad fuerzas que actúan sobre otros cuerpos. ¿Cuáles son los otros
(Fg): 105 N hacia abajo, empuje de los motores (FM): 104 N cuerpos para cada uno de los siguientes casos? a) el
hacia adelante; sustentación (FS):2x105 N hacia arriba; transbordador espacial en órbita alrededor de la Tierra b) la
resistencia del aire (Fr): 103 N hacia atrás. ¿Cuál es la patinadora que desacelera sobre el hielo c) la nave espacial
fuerza neta sobre el aeroplano? Voyager I después de rebasar la órbita del planeta Plutón
2. Una persona de 588 N de peso va en un ascensor que 9. Una fuerza de 12 N de magnitud, empuja un conjunto de
experimenta verticalmente hacia arriba una aceleración de 3 bloques agrupados horizontalmente sobre una superficie sin
m/s2 Hallar a) la fuerza normal que el piso del elevador fricción, cuyas masas son m1 = 1,0 kg m2 = 2,0 kg, m3 = 3,0
ejerce sobre los pies de la persona y b) el valor de esta fuerza kg a) ¿Cuál es la aceleración del conjunto? b) ¿Cuáles son
normal si se invierte la dirección de la aceleración las fuerzas que actúan sobre el bloque 1 y la fuerza neta que
obra sobre él. c) repita la parte b) para el bloque 2 d) repita
3. Hay cinco fuerzas todas de la misma magnitud F, que la parte b) para el bloque 3
obran sobre un objeto de masa m situado en el origen del
sistema de coordenadas. Dos de las fuerzas se encuentran a 10 Dos masas están conectadas por una cuerda ligera como
lo largo del eje x, una orientada en el sentido +X, y la otra en se indica en la figura.
-X. Otras dos fuerzas están alineadas, de modo semejante,
con el eje Y. La quinta fuerza apunta hacia la dirección +Z.
Daniel Fernández Palma / Raúl Zavala Sánchez Dinámica 93

2 kg
m1
m2 a
53°

30°
Figura 3.76 una fuerza ficticia evita la caída
Fig 3.74 Una variante de la máquina de Atwood

a) Calcular la aceleración "a"


El plano inclinado y la polea carecen de rozamiento. b) ¿Qué ocurre si el plano adquiere una aceleración mayor?
Determinar la tensión en la cuerda y la aceleración de las
masas para  = 30° y m1 = m2 = 7 kg.
11 Tres cargas eléctricas se mueven en el espacio, sin que 17 Una masa de 5kg está suspendida de un dinamómetro
sobre ellas actúe ninguna fuerza excepto las fuerzas eléctricas sujeto al techo de un ascensor como se indica en la figura.
que ejercen entre si. En un sistema adecuado de coordenadas,
se pueden descomponer algunas de las fuerzas como sigue: la
fuerza que ejerce la carga 1 sobre la carga 2 es: F21 = 3i +
1,5j – 2k y la que ejerce la carga 1 sobre la carga 3 es F31 = –
5 i + 0,5 j + k ¿Cuál es la fuerza total o neta sobre la carga 1?
12. Un bloque de masa M descansa sobre una superficie
horizontal áspera. Una cuerda inclinada hacia arriba
formando un ángulo  con la horizontal, ejerce una fuerza de
magnitud T sobre el bloque. Este permanece inmóvil. Trace
un diagrama de cuerpo libre y exprese todas las fuerzas que
actúan sobre el bloque en términos de M, g,  y T Figura 3.77 El peso varia según el movimiento

13 Se tiene un ladrillo que resbala sobre un plano inclinado a


30°. Trace el DCL para el ladrillo, en el que se vean la ¿Qué lectura dará el dinamómetro a) Cuando el ascensor se
gravedad, la fricción, y las fuerzas normales. Escoja un mueva hacia arriba con velocidad constante de 25 m/s b)
sistema de coordenadas y obtenga los componentes de las Cuando el ascensor desciende con velocidad constante de 25
fuerzas en forma de ecuaciones. m/s c) Cuando el ascensor acelera hacia arriba a 11 m/s2 . d)
De t = 0 a t = 2 s el ascensor se mueve hacia arriba a 10 m/s.
14. Una persona está parada en una escalera que se recarga Su velocidad se reduce entonces uniformemente a cero en los
contra un muro áspero y se apoya en el piso. Trace el dos siguientes segundos de modo que quede en reposo para t
diagrama de cuerpo libre para el sistema de hombre y = 4 s. Describir la lectura del dinamómetro durante el tiempo
escalera. No tenga en cuenta la masa de la escalera. Si el piso t = 0 a t = 4 s.
es muy liso, puede ser que la escalera comience a resbalarse.
¿Por qué?
18. Una masa m1 está sujeta a una cuerda de longitud s1 fija
por un extremo. La masa se mueve en un círculo horizontal
sobre una mesa sin fricción. Una segunda masa m2 se une a la
primera mediante una cuerda de longitud s2 y se mueve
también en círculo como se muestra en la figura.
Determinar la tensión en cada una de las cuerdas cuando las
masas giran con frecuencia de f rev/s.

m1 m2
Figura 3.75 La fricción es necesaria

15. Un cuerpo de 5 kg de masa está sujeto a dos fuerzas F1 =


12 i – 9 j y F2 = – 6 i +3 j. El cuerpo está en reposo en el s1 s2
origen en el tiempo t = 0 a) ¿Cuál es el vector aceleración del
cuerpo?. b) ¿Cuál es el módulo de su velocidad en el tiempo t Figura 3.78 rotación sincrónica
= 10 s.? c) ¿Dónde está el cuerpo en el tiempo t = 5 s ?
16. Una masa de 2 kg descansa sobre una superficie 19.- Una curva de 30,48 m de radio está inclinado como se
pulimentada que tiene una inclinación de 53° y una muestra en la figura 3.79, de modo que un automóvil puede
aceleración "a" hacia la derecha también sobre una superficie tomar la curva a 48,3 km/h, a pesar de que la carretera no
sin fricción de tal manera que la masa permanece en reposo tiene rozamiento. Demostrar en un diagrama de fuerzas que
respecto al plano inclinado (Ver figura adjunta). una componente de la fuerza normal ejercida por la carretera
sobre el coche puede proporcionar la fuerza centrípeta
94 Dinámica Daniel Fernández Palma / Raúl Zavala Sánchez

necesaria si el ángulo  es suficientemente grande 24. Una masa m se lanza de un origen de coordenadas con
Determinar el ángulo de peralte  en estas condiciones. una velocidad v = voi. Sobre la partícula actúan las fuerzas:
  
F1 = – Fo j y F2 = – bvx i . Encontrar: a (t), v (t), y r (t) .
25. Un electrón entre dos placas paralelas cargadas, en
posición vertical, experimenta una fuerza constante,
R perpendicular a las placas. Suponga que el electrón cuya
masa es 9x10-31 kg, se mueve en dirección horizontal con vx
 = 106 m/s y entra a una región entre las dos placas. Esta
región tiene una longitud horizontal de 1 cm y mientras está
Figura 3.79 pista peraltada allí, el electrón experimenta una fuerza de 10 -18 N a)
¿Cuánto tiempo dura el electrón en esa zona? b) Después de
haber pasado por la zona. ¿Cuál es la componente vertical de
20. Un cuerpo de 2 kg de masa es lanzado hacia arriba con un su velocidad?.
plano inclinado 37° con la horizontal. La velocidad inicial es
20 m/s y después de ascender lo mayor posible retorna a su y
punto de partida con una velocidad de 6 m/s. Calcular: a) La
distancia que recorrió a lo largo del plano b) El coeficiente de
fricción me
x v
21. Una masa M que puede desplazarse en un plano a lo 0
largo de un cateto "a" de un triángulo rectángulo, es atraído
por las masas m1 y m2 situados en los vértices con fuerzas
proporcionales a las distancias respectivas (independientes de 1 cm
las masas); siendo k la constante elástica de proporcionalidad
(N/m). Hallar la posición de equilibrio de la masa M y la
fuerza que ejerce sobre el cateto "a" Figura 3.82 electrón proyectil

m2
b 26. Una variante de la máquina de Atwood consiste en dos
masas conectadas por una cuerda sin masa y una polea sin
M masa y sin fricción, pero con las masas colocadas sobre
a
planos de distintas inclinaciones. No hay fricción entre masas
y planos. Determine la aceleración, velocidad y desplazamiento
de cada masa como función del tiempo.
m1

Figura 3.80 Atracción magnética y fuerza de contacto


m2
m1
22. Determinar la fuerza que será necesario aplicar para
levantar la masa m1 (m1 > m2)
1 2

. Figura 3.83 ¿a donde se mueve?

27. Considérese un arreglo de tres poleas que se muestran en


la figura. Las tres masas m1, m2, y m3 tienen los valores: 2,00;
4,00 y 5,00 kg respectivamente. Ninguna polea tiene fricción y
F ningún cable masa. ¿Cuáles son las tensiones en todos los
m2
cables y las aceleraciones de las masas?
m1

Figura 3.81 elevando las dos masas

23. Determine la velocidad teórica máxima en km/h, que m1 m3


pueda alcanzar el automóvil en una distancia de 50 m,
partiendo del reposo, sabiendo que el coeficiente de fricción m2
entre el pavimento y las llantas es 0,8. suponer que el 60%
del peso está distribuido en las llantas delanteras y que el
Fig. 3.84 aceleraciones distintas
40% restante en las posteriores considerar los siguientes
casos: a) Tracción en las cuatro llantas b) Tracción sólo en 28. Un trabajador debe empujar una caja de 55 kg por un
las llantas delanteras piso. El coeficiente de fricción cinética entre la caja y el piso
es k = 0,4 . ¿Cuál es la fuerza mínima que debe ejercer el
Daniel Fernández Palma / Raúl Zavala Sánchez Dinámica 95

trabajador para mantener la caja moviéndose por el piso, una r


vez que se pone en movimiento? m2
29. Una masa m descansa sobre un plano inclinado que
forma un ángulo  con la horizontal. Se le aplica una
fuerza F, que actúa en dirección horizontal. ¿Cuál es la m1
aceleración de la masa m pendiente arriba, si el coeficiente
de fricción cinética entre la masa y el plano inclinado es k?
30. Se tiene una máquina de Atwood con dos planos Figura 3.86 Fuerza centrípeta
inclinados como en la figura 3.50, pero en este caso el 36. Dos cuerpos de masas m1 y m2 están atadas por un hilo
coeficiente de fricción cinética para las superficies en contacto es que puede soportar la tensión T. A los cuerpos se les aplica
k .¿Cuál es la aceleración del movimiento. Si m1 > m2 y 1 > 2?. las fuerzas F1 = kt; F2 = 2kt (k es un coeficiente constante y t,
Suponga que al principio, m2 se mueve pendiente abajo con el tiempo) determínese el instante en el que el hilo se
velocidad vo romperá.
31. Una masa m1 descansa sobre otra masa m2 que a su vez 37. La figura muestra un hombre de 72 kg impulsándose
descansa sobre una superficie horizontal sin fricción. Se fija hacia arriba utilizando la fuerza de sus brazos. La fuerza
una cuerda ligera a m2, para tirar de ella con una fuerza F. a) ejercida por el hombre es tal que su presión sobre el sillón en
Calcule la aceleración de cada cuerpo cuando la superficie el que se apoya disminuye hasta 400 N. La masa del sillón es
entre los dos no tiene fricción b) Determine la aceleración de de 12 kg. Si g = 10 m/s2 ¿qué aceleración tendrá el hombre y
cada cuerpo, cuando la superficie entre ellos es lo el sillón?. ¿Cuál será la fuerza total que actúa sobre la polea?
suficientemente áspero para asegurar que m1 no se resbale
sobre m2. c) ¿Cuáles son la magnitud y dirección de las
fuerzas de contacto que ejerce el cuerpo inferior sobre el
superior, suponiendo que este resbala sobre el inferior con un
coeficiente de fricción cinética k?. d) Si la superficie entre
los dos cuerpos es tal que el superior resbala sobre el inferior
bajo la influencia de la fricción cinética, con el coeficiente
respectivo k, calcule la aceleración de cada cuerpo.
m1
F
m2

Figura 3.85 la fricción moviliza la masa superior


Fig 3.87 ¿Venciendo a su propio peso?

32. Suponga que la fuerza de resistencia sobre un paracaídas


está expresada por la 38. Un avión reactor de masa m que desarrolla un empuje F,
se desplaza desde el lugar de despegue por una recta, cuya
ecuación FD = ½ ACDv2 . El área efectiva inclinación respecto a la horizontal está dada por el ángulo .
¿A qué distancia del lugar de despegue se encontrará el avión
del paracaídas es A = 40 m2, y la densidad  del aire es 1,25 al cabo de un tiempo T después del despegue?. Despreciar la
kg/m3. El paracaidista cuyo peso es 80 kgf, ve que su variación de masa del reactor y la resistencia del aire.
velocidad terminal v es de 5 m/s ¿Cuál es el coeficiente de
resistencia CD ?
33. Un hombre de 65 kg de masa está parado en la orilla de 39. Entre dos barras iguales de masa M se coloca una cuña de
un tiovivo de 5,5 m de radio. El aparato gira a 6,0 rev/min. masa m con ángulo . Determínese las aceleraciones de los
¿Cuáles son la magnitud y dirección de la fuerza neta cuerpos si no existe rozamiento.
ejercida sobre el hombre?
34. Un automóvil toma una curva cuyo radio es de 100 m. La
carretera está peraltada a un ángulo de 15° y el coeficiente m
de fricción entre neumáticos y pavimento es 0,22. Calcule las 
velocidades máxima y mínima que debe tener el automóvil M M
para permanecer en la carretera, sin derrapar hacia arriba o
hacia abajo del peralte.
35. Una masa m1 cuelga de una cuerda que se pasa por un Figura 3.88 Acción de la cuña
agujero en una mesa lisa y sin fricción. En el otro extremo de
la cuerda se fija la masa m2 a la distancia r del agujero de la
mesa y se mueve en órbita circular alrededor del agujero 40. Una carga está colgada del extremo de un hilo, sujeto por
¿con qué velocidad debe girar m2 para que r se mantenga el otro extremo de una pared. La carga oscila en torno al
fijo? punto de apoyo del hilo sobre la polea fija en una barra de
masa M. Esta última puede deslizar sobre el plano horizontal
sin fricción. En el instante inicial se desvía la carga en un
ángulo  respecto a la vertical y se la deja libre. Determinar
la aceleración de la barra, si el ángulo formado entre el hilo y
96 Dinámica Daniel Fernández Palma / Raúl Zavala Sánchez

la vertical, no cambia al moverse el sistema. ¿Cuál es la masa mesa. Las poleas tienen masa despreciable y el sistema está
de la carga? inicialmente en reposo.

m1

M 
m2

Figura 3.89 Fuerza ficticia


Figura 3.92 Las distancias recorridas
41. ¿Cuál deberá ser el coeficiente de fricción entre la barra no son las mismas
homogénea y el piso, para que pueda mantenerse en la
posición que se indica en la figura?. La longitud del hilo AB
es igual a la longitud de la barra. a) encuentre la relación que debe existir entre las distancias
d1 y d2 recorridas por m1 y m2 cuando el sistema empieza a
moverse. b) encuentre una expresión para la tensión en la
A cuerda
45. Una cuerda se pasa a través de un tubo de vidrio liso.
90° B Dos cuerpos de masas M y m se atan a sus extremos con M >
m . como se muestre en la figura 3.93, el cuerpo m se hace
girar alrededor del tubo en un circulo horizontal, de manera
que el cuerpo M ni sube ni baja. El periodo del movimiento
circular es T a) ¿Cuál es el ángulo entre la cuerda y el tubo?
B) exprese la longitud libre de la cuerda L en función de T,
M, m y g c) exprese T en términos de g y h, donde h es la
Fig 3.90 Equilibrio por fricción distancia vertical desde la parte superior del tubo hasta el
cuerpo de masa m

42 ¿En la figura 3.91 hasta qué valores del ángulo  el cubo


sujeto de la arista se mantendrá en equilibrio? El coeficiente
de fricción entre el cubo y la pared es 
h

R m

Figura 3.91 Equilibrio por contacto


Figura 3.93 La fuerza centrífuga aleja a
los cuerpos del eje de rotación
43 En el ecuador una persona tiene un peso efectivo
ligeramente menor que el que tendría si la Tierra no girase.
Esto se debe a la fuerza centrífuga ficticia; en el ecuador esta
fuerza actúa en dirección opuesta a la de la verdadera fuerza
gravitatoria. a) muestre que la fuerza centrífuga ficticia F c
que observa una persona de masa M en la latitud  tiene de
módulo (42RM/T2)cos y apunta hacia fuera en una
dirección perpendicular al eje de rotación de la Tierra; donde
R es el radio de la Tierra y T su periodo de rotación. b)
construya un diagrama a escala que muestre la verdadera
fuerza gravitatoria W = Mg y la fuerza centrífuga Fc que
actúan sobre una persona en la latitud  c) Encuentre una
expresión para el peso efectivo de una persona en la latitud
.
44. Como se muestra en la figura un cuerpo de masa m1
descansa sobre la superficie horizontal sin rozamiento de una

You might also like